ChaseDream

标题: OG10语法268题及GWD22语法总结日记,监督自己并欢迎大家多给意见 [打印本页]

作者: elinaqu    时间: 2007-5-16 00:32
标题: OG10语法268题及GWD22语法总结日记,监督自己并欢迎大家多给意见

为了监督自己能够更加注重质量的总结OG语法,特开此帖,自第二楼起将会一楼总结一道OG10的语法题,希望大家多给意见,多多补充,谢谢!

* 为了环保没有另开新??话言?吹囊桓鑫赐瓿傻挠锓ㄗ芙崽??谋晏夂湍谌菅赜昧恕?lt;/p>

声明:很多语法点的总结摘录自前辈NN的讨论贴,特此感谢!


今天开始(27Aug,2007)开始GWD22的语法总结,再次声明很多语法点的总结摘录自前辈NN的讨论贴,特此感谢!!
[此贴子已经被作者于2007-8-28 22:42:55编辑过]

作者: elinaqu    时间: 2007-8-4 18:19

1.   The Wallerstein study indicates that even after a decade young men and women still experience some of the effects of a divorce occurring when a child.

(A)    occurring when a child

(B)    occurring when children

(C)   that occurred when a child

(D)   that occurred when they were children

(E)    that has occurred as each was a child

1.        That引导的定语从句修饰和ing格式分词修饰名词的区别(为什么that occurred正确而occurring就不对呢)

观点一(一次性动作和反复性动作)

that+从句修饰名词表示一次性的动作。v-ing修饰名词表示反复性的动作

现在分词表示动作的反复、正在进行和延续,用在这里明显不合适,况且后面还有时间状语when,只能是过去分词表示一次性发生的动作。

现在分词做定语,有三个作用,其一,相当于进行时态表示的正在进行的动作,这个好理解;其二,相当于一般时态所表示的任何时候都能发生的动作,eg:A man living in town rarely sees cows.要注意:做定语的现在分词和谓语动作必须表示相同的时间,否则只能用定语从句,不能用现在分词。eg:I muse find out the child who broke my door.(not breaking my door)如果是最后这一句所示的规则。那么该句使用that occured修饰divorce是非常合理的。(张道真语法的解释)

1  - 白勇语法里此问题的例句及一段解释

In Egypt in the late Palaeolithic period, the climate changed, pastures became deserts, and the inhabitants were forced to withdraw to the land bordering the Nile from their hunting grounds. .

A. inhabitants were forced to withdraw to the land bordering the Nile from their hunting grounds. .

B. inhabitants had been forced to withdraw from their hunting grounds to the land that bordered the Nile

C. inhabitants were forced to withdraw from their hunting grounds to the land bordering the Nile.

D. inhabitants having been forced to, withdraw from their hunting grounds to the land that bordered the Nile

E. inhabitants withdraw, because they are forced to, from their hunting grounds to the land bordering the Nile.

C为正确选项,the land bordering the Nile 合理的逻辑关系应为:后者作为前者的动作具有客观性,多次性和重复性,无明确时间概念,应使用ing形式做定语强调这一逻辑关系,定语从句做定语强调动作的一次性和具体时间下的具体行为。

2  OG10-80

Salt deposits and moisture threaten to destroy the Mohenjo-Daro excavation in Pakistan, the site of an ancient civilization that flourished at the same time as the civilizations in the Nile delta and the river valleys of the Tigris and Euphrates.

A. that flourished at the same time as the civilizations

B. that had flourished at the same time as had the civilizations

C. that flourished at the same time those had

D. flourishing at the same time as those did

E. flourishing at the same time as those were

OG解释flourishing ongoing 的嫌疑。对于过去发生的某个动作,ETS喜欢用that did

观点二(限定性非限定性)

that定语从句和分词的区别:分词相当于非限定定语从句,而限定定语从句和非限定定语从句的区别是局部和范围的区别,所表达的意思有很大的差异

A. 限定性修饰, 起限制约束作用:

1.that 引导的定语从句, (that只引导限定性定语从句, 前面不会有逗号)

2.由前面没有逗号的一wh开头的词(who, which...)引导的定语从句,

3.with引导的短语

构成的修饰就是限定性的修饰, 起限制的作用. 就象OG1里的that定语从句. 什么样的divorce? 是发生在when they were children时候的divorce, 而不是所有的divorce. 即限定

B. 非限制性修饰:

1. 由前面有逗号的一wh开头的词(who, which...)引导的定语从句

2.. 由分词短语

构成的修饰就是非限制性修饰, 起解释, 说明的作用. 就象OG1里的divorce, 如果用分词修饰就变成 divorce 的定义(definition) occuring when a child, 即解释, 说明.

1前辈NN讨论贴里关于限定非限定的强贴

1He asks the girls who is in red clothes to form a team.

2He asks the girls with red clothes to form a team.

3He asks the girls, who is in red to form a team.

4He asks the girls, dressing in red clothes to form a team.

1,2句是限定的,可以想象成体育馆中有一大群女孩儿,有穿红的,穿绿的,有黄的等等,这群女孩子中穿红的那一部分组成一个队。此时这个穿红的修辞成份是对女孩子的群体起限定作用,是局部概念。

23句是非限定的,可以想象成体育馆中有一大群女孩子,所有的女孩子一个不落地组成一个队,这些女孩子共有一个特点,全都是穿红的。此时穿红的修辞成份是对女孩子的群体不作限定,只表示这一个群体的特点。

不加逗号的定语从句是限定的,加逗号的定语从句是非限定的。 With限定,-ing-ed分词不限定 

2.    用代词they were清楚指代先行词,什么时候该有代词they.,什么时候该省略?

从句中主语的省略问题

从句中如果有省略主语,其逻辑主语指向它修饰的部分的中心词。要考虑他是否合理

The complete form of choice A is "occurring when (divorce was) a child" . Note:The clause is 'A divorce occured when (it was)a child', the logic subject (which is the subject of the clause) and link verb are omitted. So if you want to restore the omitted part, you have to use the subject of the sentence plus be. It is a rule, so memorize it. Of cource the meaning of this clause is absurd, so A is wrong. Pls refer to 《新编英语语法,章振邦》P1153chapter36,无动词分句。

--主从主语一致才可以省略

3.结构和逻辑上的错误

Choice A incorrectly introduces the when... phrase with occurring, thus illogically making divorce the grammatical referent of when a child;

Divorce occurred when (divorce) a child.

4.   单复数一致, a child,children, 另外Each 指代单数,译为每一个,各自的,与文中的复数不一致)

5.为什么有men and women ,effects children, they 等复数 divorce 却用单数呢.

一个群体如果指一个共有特征的时候,这个特征可以用单数。

6.   时态错误.--为什么这里现在完成时错误?什么时候用过去时什么时候用现在完成时?

比较过去时与现在完成时的区别

1)过去时表示过去某时发生的动作或单纯叙述过去的事情,强调动作;现在完成时为过去发生的,强调过去的事情对现在的影响,强调的是影响。

2)过去时常与具体的时间状语连用,而现在完成时通常与模糊的时间状语连用,或无时间状语。 
3)现在完成时可表示持续到现在的动作或状态,动词一般是延续性的,如live, teach, learn, work, study, know. 过去时常用的非持续性动词有come, go, leave, start, die, finish, become, get married等。







[此贴子已经被作者于2007-8-6 0:34:31编辑过]

作者: elinaqu    时间: 2007-8-4 18:43

7。When为什么不能取代as, when什么时候可以取代as? When as的区别是什么?when可不可以引到短语?

as 可能强调"随着", 表示伴随动作, 好像还是一个进行中的过程,

whenwhileas显然都可以引导时间状语从句,但用法区别非常大。   
一、when可以和延续性动词连用,也可以和短暂性动词连用;而whileas只能和延续性动词连用。
    
二、when从句的谓语动词可以在主句谓语动作之前、之后或同时发生;whileas从句的谓语动作必须是和主句谓语动作同时发生。
    
1
.从句动作在主句动作前发生,只用 when
    
2
.从句动作和主句动作同时发生,且从句动作为延续性动词时,whenwhileas都可使用。
    
3
.当主句、从句动作同时进行,从句动作的时间概念淡化,而主要表示主句动作发生的背景或条件时,只能用 as。这时,as常表示随着……”一边……,一边……”之意。
   

    
三、when用于表示…………”的句型中(指过去的事情)。
    
sb
had hardly(=scarcely done sthwhen...=Hardly Scarcely had sbdone sthwhen

只能用when 的句型:

放句中,主句是进行时,从句是短暂动作,表示这时突然

从句是短暂动作:When I got up, I heard the bell ring. 当我起床的时候,我听见铃响了。

只能用while的句型:

1. 表示对立予盾的对比,这时while相当于butI am poor while you are rich.我很穷,你却很富有。 

2. 表示趁着还来得及的时候,赶快.... 趁热打铁.Strike while it is hot. 

只能用as的句型:

属下列情形时,只用as, 而不用whenwhile

A) 用于表示同一个人的两种动作交替进行,"一边……,一边……"。如:

He looked behind from time to time as he went forward.

B)表示两个同步发展的动作或行为,译为"随着……"。如:

As children get older, they become more and more interested in the things around them. 

C)表示两个短促行为或事件几乎同时发生。如:

Just as the flying worm hit her face, she gave a loud cry.

D) 后接名词表示某一年龄段时,as。如:

As a young man, he was active in sports. 

as like的区别

1. 表示象...一样时, as接从句, like 接短语
Do everything as I do. 象我一样做。
He is/looks like his mother.
他长得像他妈妈。

2. as当介词接短语时,表示作为...不是象...一样的意思.like 当动词时,是喜欢的意思,不要搞混为象...
As a League Member,I should take everything in the lead.
作为一名团员,我应该起带头作用。

while的用法
    
一、while意为……时候”,指一段时间,不能用来表示一个时间点。
    
While the discussion was still going on, George came in.
当讨论还在进行时,乔治走了进来。
   

二、while意为;然而”,表示转折。
    
There're plenty of rain in the southeast, while there's little in the northeast.
东南部雨量充足,而西北部则很少下雨。
    
三、while意为虽然;尽管”,表示让步。
    
While I admit his good points, I can see his shortcomings.
尽管我承认他的优点,但我还是能看到他的缺点。
   

四、while意为只要”,表示条件。
    
We can surely overcome these difficulties while we are closely united.
只要我们紧密地团结一致,一定能克服这些困难。
    
五、while从句中的省略。
    
while从句中的主语与主句的主语一致,且含有be的某种形式时,从句中的主语连同be可同时省略。 
While in
London, he studied music and painting.
在伦敦时,他学习音乐和绘画。
    
六、while可用作名词,意为一会儿;一段时间
    
作名词用时,主要用于短语中: after a while “过了一会儿”; all the while “一直,始终”; a short / little while ago “刚才”; once in a while “偶尔,间或”; wait / rest(for) a while “ / 休息一会儿.

As的用法

一、 as作连词的用法:

1. "……的时候",引导时间状语从句。注意与when while的用法区别。(同上)

2. as=since, "既然""由于",引导原因状语从句,常用来表示已为人们所知或显而易见的原因或理由。如:

As / Since you're not feeling well, you may stay at home.

3. as=in the way that, """按照……的方式",引导方式状语从句。如:

He speaks English as Americans do.

4. 用于as...as...not so/ as...as...,前一个as是副词,后一个as是连词,引导比较状语从句。如:

I don't speak English so/ as well as he does. 

5. "虽然""尽管",引导让步状语从句。常用倒装语序,adj. /adv. / n.+ as +主语+谓语+主句。如:

Child as he is, he knows a lot. (child前不用冠词)

二、 as作关系代词,引导定语从句,"正如""这一点"解。如:

He is very careful, as his work shows.

另外,当先行词被the same, such等词修饰时,常用as来引导定语从句。如:

Such a clever boy as he can learn anything quickly. 

三、 as作介词的用法:

as引导的介词短语大多用作状语,as译为"作为";少数情况可引起宾语补足语。如:

As a League member, I'll take the lead in everything. (状语)

She works as a doctor.  (状语)

They have me as one of their own children.(引起宾语补足语)

When后边接名词的一个例子(人还有疑问??)

When still a boy of six, Bob was sent away from home. (章振邦 p1155)

8divorce后面的that能不能省掉。

定语从句中that的省略

that引导的定语从句并且that做主语,那么that不能省略

9A of B 结构的修饰问题

--- A of B+that定语,其修饰的主体是B

--- A of B+分词,分词有可能是修饰A ,也有可能是修饰 B

跳跃和就近都有可能,不能武断哪种肯定对.OG1就是明显的就近修饰,OG254则是跳跃修饰.

不能把修饰方向作为判断答案标准,可能从其他语法点着手.. A of B结构,除非有主谓一致的提示,不然只能逻辑判断。

*one of Ns thatthat后面的动词用复数

the only one of Ns that that后面的动词用单数


[此贴子已经被作者于2007-8-4 23:08:20编辑过]

作者: elinaqu    时间: 2007-8-4 22:55

2. Since 1981, when the farm depression began, the number of acres overseen by professional farm-management companies have grown from 48 million to nearly 59 million, an area that is about Colorado's size.

A. have grown from 48 million to nearly 59 million, an area that is about Colorado's size

B. have grown from 48 million to nearly 59 million, about the size of Colorado

C. has grown from 48 million to nearly 59 million, an area about the size of Colorado

D. has grown from 48 million up to nearly 59 million, an area about the size of Colorado's

E. has grown from 48 million up to nearly 59 million, about Colorado's size

1. 主谓一致the number of + 复数名词做主语,后面的动词用单数。Since + 时间,是现在完成时的标志。

2. 简洁:定语从句的省略

a.that做定从中主语,不可省。there is a girl that(who) has green hair..

b that做定从中宾语,可省,也可不省。there is a book (that) you have not read.

c that be/which be引导的定从,能省就省。there is a book (that was) borrowed by mary yesterday.

名词+thatbe+分词/介词等一般没有分词+名词或者名词+介词等的形式简洁;不过还是需要逻辑判断的,如果是名词+that +情态动词+be+分词/介词大多数不要使用上面大额原则,否则意思改变;

* 定语从句的that: 省略 & 宾语从句的that: 不省

3.逻辑意思清晰(此题about前边要有名词)

同位语结构

同位语是GMAT的优先结构;当补出概括性名词的时候更加清楚。介词短语前面一定要有所修饰的名词

4.习语: 

from x to y ; 改成 from x up to y 错!

5. 所有格错:

a..双重所有格必错,The A of B’s

b. A 's主要用于表示有生命的东西,另外表示时间(today 's papers)、人组成的集体(government 's policy)、国家城市(Beijing 's parks)、某些机构(hotel 's entrance) 的名词也可用

c. A of B , B 通常为物.但是这一点在 GMAT中不要作为第一位

朗文英语语法(名词-所有格):一般不使 'S与无生命的东西连用。但与地理、机构有关的,可以用 'S / S' of 结构,例如 Amercia's policy, the European Economic Community's exports

下列情况一般用 'S / S':地名+最高级,教会和教堂,和时间有关的,值多少钱,固定说法。例如 New York's tallest skyscraper, St Paul's Church, a day's work, an hour's delay, a month's salary, two days' journey, twenty dollars' worth of gasoline, 固定说法at arm's length, the earth's surface

关于's 这个问题,ETS考的非常灵活, 很多没有生命的都加了, 建议不要把除了双重所有格以外的和's相关的东西作为排除点.

6.        Nearly

nearl否定形式通常用hardly / scarcely / barely 来代替。例如:
I almost / nearly didn't get up in time.

I hardly / scarcely / barely got up in time.



[此贴子已经被作者于2007-8-4 23:43:41编辑过]

作者: elinaqu    时间: 2007-8-6 00:20

3. Some bat caves, like honeybee hives, have residents that take on different duties such as defending the entrance, acting as sentinels and to sound a warning at the approach of danger, and scouting outside the cave for new food and roosting sites.

A. acting as sentinels and to sound

B. acting as sentinels and sounding

C. to act as sentinels and sound

D. to act as sentinels and to sound

E. to act as a sentinel sounding

1 平行结构:

此题大平行里小平行

A.平行结构的形式对称 ==现在分词和不定式不对称。

B.  平行结构可以有多个层次,此处就是  such as A, B1 and B2, and C1.  其中 A,B,C大平行, B1 and B2;  是小平行。

2  单复数一致

Residents that … acting as + 复数名词对应,表示同一内涵的对象单复数一致。

3 保持原句意思

平行结构改成分词修饰结构会改变原句的意思。

4 Such as / Like 用法:

A like B.      A B是同类的事物进行比较。

Asuch as B , C          B 和C是对A所包含事物的列举。


[此贴子已经被作者于2007-8-6 0:27:34编辑过]

作者: нандин    时间: 2007-8-6 15:29

楼主MM辛苦了 顶一下


作者: elinaqu    时间: 2007-8-6 17:03

4. The only way for growers to salvage frozen citrus is to process them quickly into juice concentrate before they rot when warmer weather returns.

A. to process them quickly into juice concentrate before they rot when warmer weather returns

B. if they are quickly processed into juice concen­trate before warmer weather returns to rot them

C. for them to be processed quickly into juice concentrate before the fruit rots when warmer weather returns

D. if the fruit is quickly processed into juice concen­trate before they rot when warmer weather returns

E. to have it quickly processed into juice concentrate before warmer weather returns and rots the fruit

1. 平行结构:平行结构的形式对称  the only way to … is to …

2 单复数一致:代词要和他指代的名词的数一致。 Them/it 的选择

Citrus 单复数都可以,这里用单数,因为指一类

it is better to use the singular in referring to an anatomical feature common to an entire species;这是etsog167

3 代词正确答案的标志:当前面有多个名词可以指代的时候,通常补出概括性的名词来去掉代词,防止歧义。

4.句子逻辑意思: when 引导的时间状语从句只强调动作发生的时间上的同步,没有强调因果的意思。表示因果/目的,可以用不定式。A and B A/B 是两个连续的动作的时候,可以存在强调因果的意思在。   

5   习惯用法Before / after …. When 连用不符合语言习惯

6Rot 的用法 既可以是及物动词又可以是不及物动词

6Rot 的用法 既可以是及物动词又可以是不及物动词


[此贴子已经被作者于2007-8-6 17:05:49编辑过]

作者: elinaqu    时间: 2007-8-6 17:03

5. Carbon-14 dating reveals that the megalithic monu­ments in Brittany are nearly 2,000 years as old as any of their supposed Mediterranean predecessors.

A. as old as any of their supposed

B. older than any of their supposed

C. as old as their supposed

D. older than any of their supposedly

E. as old as their supposedly

1.为什么要是than,不可以是as…as呢?

比较结构:
A more than B or  A as + adj+ as B. as.. as 结构的平行问题)A as + adj+ as B 的形式中, A/B 要是同类而且对称的。逻辑意思决定选择比较结构:2000 years Mediterranean predecessors. 不对等,不能用 as old

nearly predecessor 都暗示此处要选择 more .. than结构

如果要表示对等,应该是A is nearly 2000 years old, as old as any of ….

这道题用比较级的关键在于划线部分前面是2,000 years ,如果不是比较级,前面不能放 2.000 years

2 修饰:形容词修饰名词性短语, 副词修饰动词/形容词, Mediterranean predecessors. 是名词性短语,用形容词修饰

3. 保持原句意思平行结构改成分词修饰结构会改变原句的意思。


[此贴子已经被作者于2007-8-6 18:23:28编辑过]

作者: elinaqu    时间: 2007-8-7 19:15

6. In virtually all types of tissue in every animal species, dioxin induces the production of enzymes that are the organism's trying to metabolize, or render harmless, the chemical that is irritating it.

A. trying to metabolize, or render harmless, the chemical that is irritating it

B. trying that it metabolize, or render harmless, the . chemical irritant

C. attempt to try to metabolize, or render harmless, such a chemical irritant

D. attempt to try and metabolize, or render harm­less, the chemical irritating it

E. attempt to metabolize, or render harmless, the chemical irritant

1.       抽象名词优于动名词做名词性结构----所有格的宾语。

Attempt 好于 trying 做所有格的宾语。动名词强调动作本身,抽象名词强调对应动作的结果。

2.  并列结构:

A  is  B 结构中, A/B 要平行。enzymes that are the organism's trying结构中 enzymes 要和 are 后面的成分对等,即名词 are 名词此时宾语不能用动名词)

3. 特殊结构:

OG 解释The phrase the chemical irritant is also the most concise and precise conclusion for the sentence because it clearly refers to the dioxin mentioned earlier.

In virtually all types of tissue in every animal species, dioxin induces the production of enzymes that are the organism's attemptattempt to metabolize, or render harmless, the chemical irritant

render harmless, the chemical irritant   ==render the chemical irritant harmless 宾语放在后面。render为"使得"的意思(后接受词+受词补语 )

chemical irritant->dioxin(受词, O.)

harmless->受词补语(O.C.)

4. virtually modifies all 而不是修饰 all types of  tissue

5.   简洁: attempt to try 不简洁

5.   简洁: attempt to try 不简洁

6  词法 try + that … 结构错误, try to do sth.

7.  主语和宾语的数的一致:宾语为具体名词时,通常要求主语和宾语数的一致。宾语为抽象名词时,主语和宾语的数可以不一致。enzymes that are the organism's attempt    Attempt 此处表示抽象的动作,可以用单数。

7.  主语和宾语的数的一致:宾语为具体名词时,通常要求主语和宾语数的一致。宾语为抽象名词时,主语和宾语的数可以不一致。enzymes that are the organism's attempt    Attempt 此处表示抽象的动作,可以用单数。

看贴后本题最到收获---前辈NN的宝贵意见

注意主考点就好了,OGSC每一题都有明显的主要的考点,根据这些考点很快就可以把题做出来。ETS绝对不会单考某一个小小的单复数的考点,并且将这个考点作为主要解题依据的。偶们是为了拿分而复习的,这些小细节放过吧。P.S.我总结下来,考单复数的题无非两种考法:1 主谓搭配 2 指代。本题两者皆不是,不要太在意。所以本答案原句为: attempt to metabolize, or render the chemical irritant harmless


[此贴子已经被作者于2007-8-7 19:21:39编辑过]

作者: crowkiller    时间: 2007-8-7 19:20

Elina 辛苦了!

坚持住!加油!


作者: elinaqu    时间: 2007-8-7 19:23

7. Dr. Hakuta's research among Hispanic children in the United States indicates that the more the children use both Spanish and English, their intellectual advantage is greater in skills underlying reading ability and nonverbal logic.

A. their intellectual advantage is greater in skills underlying reading ability and nonverbal logic

B. their intellectual advantage is the greater in skills underlaying reading ability and nonverbal logic

C. the greater their intellectual advantage in skills underlying reading ability and nonverbal logic

D. in skills that underlay reading ability and nonver­bal logic, their intellectual advantage is the greater

E. in skills underlying reading ability and nonverbal logic, the greater intellectual advantage is theirs

1 平行结构:

the more … the more… 注意前后结构中的主语、谓语的省略。the more...the more...结构其实是一个从句+主句的结构:
1. 
第一个the more...相当于一个原因状语从句是从省略了表示原因的连词as等进化而来的(也可理解成是省略了if的条件状语从句); 第二个the more...引导的是主句
eg.As a mammal's skin is thicker(
从句), it has less hair(主句). 转换为:
The thicker a mammal's skin is(
从句), , the less hair it has(主句). 

2.两个the都是副词(而非冠词), 表示程度并修饰后面的形容词比较级. (前后两个the的意思在 韦氏字典解为: the: adv. 2 a : to what extent <the sooner the better> b : to that extent <the sooner the better>)

the more...the more...结构的三个原则(参自[托福语法填空要点])
1. the more
后面的名词必须使用冠词或所有格
如本题的the more the children..., the greater their ...

2. 后面的谓语如果是be动词的话可以省略这一点对于前后两个都适用. (特别当主谓语时it is常同时省略.) 
eg.What size box do you want?---The bigger, the better. 
就是的The bigger it is, the better it is的省略结构上面的252the greater...引出的主句显然省略了Be动词is, 表语是greater, 提到句首加副词the表示程度构成并列的

3. 第二个the more后面可以使用倒装而第一个后面却不行. (因为只有主句才能倒装,从句绝不能倒装如果继续深究第二个the more后面什么时候用倒装时可认为如果主语长,谓语动词短时为避免头重脚轻主谓语倒装.) ----不懂

上面的252题如果要补出谓语的话, the greater is...结构较好但是Official GuideD的解释是:The phrase [the more the children] should be completed by a parallel phrase that begins with a comparative adjective and a noun phrase, as in [the greater their...] advantage. Only C correctly completes the structure with a parallel phrase. 

2  词法: underlie / underlay

underlie vt.位于...之下, 成为...的基础;

underlay vt 把(某物)放到另一物的下面.从下面支撑

underlying cause/principle/problem etc. the cause, idea etc that is the most important, although it is not easily noticed

the underlying causes of her depression                                 

There is an underlying assumption that younger workers are easier to train.


[此贴子已经被作者于2007-8-7 19:24:39编辑过]

作者: elinaqu    时间: 2007-8-7 19:28

8.  Lacking information about energy use, people tend to overestimate the amount of energy used by equipment. such as lights, that are visible and must be turned on and off and underestimate that used by unobtrusive equipment, such as water heaters.

A. equipment, such as lights, that are visible and must be turned on and off and underestimate that

B. equipment, such as lights, that are visible and must be turned on and off and underestimate it when

C. equipment, such as lights, that is visible and must be turned on and off and underestimate it when

D. visible equipment, such as lights, that must be turned on and off and underestimate that

E. visible equipment, such as lights, that must be turned on and off and underestimate it when

此题平行为最大

1. when从句/词组没有先行词的时候, 直接修饰主语

参考OG10-.1when从句/词组没有先行词的时候, 直接修饰主语
occurring when a child…
people when used by …

2 平行:

overestimate the amount of energy used by visible equipment  and underestimate that used by unobtrusive equipment, such as water heaters.

A>n.+ used by visible equipment n.+ used by unobtrusive equipment 平行。

3.  单复数一致:

equipment, such as lights, that is visible

that 是限定定语从句,不能修饰前面直接接的名词lights.只能将such as lights 理解为插入同位语,that 从句修饰 equipment.


[此贴子已经被作者于2007-8-7 19:29:32编辑过]

作者: elinaqu    时间: 2007-8-7 19:36

我觉得第9题很难

9. Astronomers at the Palomar Observatory have discovered a distant supernova explosion, one that they believe is a type previously unknown to science.

A. that they believe is

B. that they believe it to be

C. they believe that it is of

D. they believe that is

E. they believe to be of

1.为什么不能是that

当先行词做定语从句的宾语,that 可以省略 one ( that ) they believe to be …….

2         代词:it 多余。

One that they believe it to be / One that they believe that it is ………  

believe 后面已经有逻辑宾语 one ,it 多余

3  平行结构:
A is B A/B 要平行对等,

此处 one(explosion ) is a type previously unknown to science.  Explosion 不能是 a type. 可以是one of a type

4.  词法believe X to be Y / believe that ……

观点一to beis在表示时间方面要好. 因为is是他们一直都这么认为, 时间概念太明确, 然而, TO BE就有模糊性, 所以这个题目里头OGIn the context of this sentence, the infinitive to be is more appropriate than the limited present-tense is in referring to an event that occurred long ago but has been discovered only recently.

观点二

这题偶的理解比较单纯,explosion, the one they believe to be of a type.  定语从句,
   
先行词还原到定语从句结构中就是: they believe the explosion to be of a type..... 

根据believe 的两种用法:
believe that  ..... 
believe sth to be sth.....这里附和第二种用法。很自然的排除用限定性结构,(包括没有that, 直接用is 的结构。)

不过语言有很大的约定俗成的部分,即某种结构本身就带有某种含义。最好的办法是记住,形成习惯性思维,直到成为语感。看到某种结构就觉得应该是这个意思。 

下列動詞接toV時總是用被動
allege, assume, claim, consider, deem, feel, find, know..., believe, ...
他們表示言談,想法或發現時,後面常接 to be  to have Vpp

that如果引导宾语从句, 一定不能省略.

that如果引导定语从句: 1. 如果that在从句里做主语, 不能省略; 2. that在从句里做宾语, 省略.

但上面的2OG里有反例. 243就是that做后面的宾语而不省略的. 这并不是ETS自相矛盾, 而是补出that避免歧义. 所以, 没有绝对的规律, 只有相对的正确, 因为语言的魅力在于灵活


[此贴子已经被作者于2007-8-7 19:48:27编辑过]

作者: elinaqu    时间: 2007-8-7 19:46

10. However much United States voters may agree that there is waste in government and that the government as a whole spends beyond its means, it is difficult to find broad support for a movement toward a minimal state.

A. However much United States voters may agree that        

B. Despite the agreement among United States voters to the fact

C. Although United States voters agree

D. Even though United States voters may agree

E. There is agreement among United States voters that

1 宾语从句的 that 不能省略。而且此处要和后面的 that 平行

2  Run-On 结构一个句子里不能同时出项两个主句。

3  情态动词不能随便省略 =è may 不能省。

4  词法:  the agreement … to the fact there is  不对, the fact 没有办法 agreement..

5  保持原句意思: there be 结构或者改变动词 agree agreement 不好

6 However much的用法

在英语中在很多情况下however much= very much 是修饰动词的,由于very much 只能放在句尾而不能放在句首为加强语气,所以把however much放在句首。

No matter how united states voters may agree MUCH that there...
表示:美国选民多么地同意.... 而不是说:无论多少个选民同意。看下面这个例子:
However much Mr. Robin may agree that there is waste in government and that the government as a whole spends beyond its means, it is difficult to find broad support for a movement toward a minimal state.


[此贴子已经被作者于2007-8-7 19:47:31编辑过]

作者: elinaqu    时间: 2007-8-7 19:49

11. Based on accounts of various ancient writers, scholars have painted a sketchy picture of the activities of an all-female cult that, perhaps as early as the sixth century B.C., worshipped a goddess known in Latin as Bona Dea, "the good goddess."

A. Based on accounts of various ancient writers

B. Basing it on various ancient writers' accounts

C. With accounts of various ancient writers used for a basis

D. By the accounts of various ancient writers they used

E. Using accounts of various ancient writers

1  句首的修饰结构要考虑修饰结构的逻辑和意思上的合理性:分词做句首的修饰结构,其逻辑主语指向后面的句子主语。此时要考虑是否合理,可将前面的修饰结构还原来判断。 === scholar were based on accounts of various ancient writers 不合理

2  词法: Based on….  通常 GMAT Base 都使用被动态, based on.

3  代词指代不明确: B 中的 it scholars base it(意思为their painting work)on various ancient writers' accounts,如果指a sketchy picture意思不太对,所以不知道it指谁。

代词总的来讲应根据逻辑意思决定指代对象. OG11里的they若根据逻辑来判断, 有明确的指代对象, 即主句的主语. 但当ETS要说一个选项是错的时候, 即使逻辑意思清楚, 也被认作指代模糊. 所以完全依靠代词指代模糊来决定答案不可取. 一般我只把它作为最后一招. 有其他区别就先看其他区别.

但有三个代词指代的错误, ETS认为必错

1. 单复数不一致. 如用they指代a person

2. 在同一句句子中, 同一个代词指代不同的东西. (OG12)

3. 代词指代所有格, 而不是中心词. (OG90)

4  简洁:C with accounts of  used for a basis 语意重复。 D they used 修饰不明确,也不合理,不简洁。with + n. 在句首表示physically拥有, 相比之下, with更强调一种状态. base, compare to这类分词省略形式是应该用被动的.ETS其实偏向主动句, 因为它强调了动作发出者.


[此贴子已经被作者于2007-8-7 19:50:29编辑过]

作者: elinaqu    时间: 2007-8-7 19:52
以下是引用нандин在2007-8-6 15:29:00的发言:

楼主MM辛苦了 顶一下

谢谢鼓励,我会坚持的。


作者: elinaqu    时间: 2007-8-7 19:58
以下是引用crowkiller在2007-8-7 19:20:00的发言:

Elina 辛苦了!

坚持住!加油!

killer谢谢,其实要感谢你,是看了你发给我的OG以后看了你的总结,我才有了把它全部整理的想法,而且这里很多总结都是你发得OG力总结的和讨论贴里前辈NN们的牛贴精华。希望大家不要介意我把它paste到这里。

还有,从开这个贴的那一天起,我就决定要用OG268道题一直把这个贴盖到268楼。但是我担心的是,是不是真的能全变成自己掌握的东西,又是否会对我提高语法有帮助呢?


作者: crowkiller    时间: 2007-8-7 20:50
以下是引用elinaqu在2007-8-7 19:58:00的发言:

killer谢谢,其实要感谢你,是看了你发给我的OG以后看了你的总结,我才有了把它全部整理的想法,而且这里很多总结都是你发得OG力总结的和讨论贴里前辈NN们的牛贴精华。希望大家不要介意我把它paste到这里。

还有,从开这个贴的那一天起,我就决定要用OG268道题一直把这个贴盖到268楼。但是我担心的是,是不是真的能全变成自己掌握的东西,又是否会对我提高语法有帮助呢?

坚持就有效果. 注意一些重点题目(Excel file 中用不同颜色标出来的,解释很多很详细的)。

重要的是把知识点都抓住,不是简单的把题目都机械的完成。

加油!


作者: dennyhuang    时间: 2007-8-7 23:59

支持队友一下!加油,elina好认真啊。我第二遍过OG时也这样把每个考点写出来,没你这么细,但感觉对提高语法有很大帮助,相信你一定会掌握得更好。等我整理一下我的语法总结,也贴上来。

一起加油吧!!


作者: elinaqu    时间: 2007-8-8 16:03

12. Formulas for cash flow and the ratio of debt to equity do not apply to new small businesses in the same way as they do to established big businesses, because they are growing and are seldom in equilibrium.

A. Formulas for cash flow and the ratio of debt to equity do not apply to new small businesses in the same way as they do to established big businesses, because they are growing and are seldom in equilibrium.

B. Because they are growing and are seldom in equilibrium, formulas for cash flow and the ratio of debt to equity do not apply to new small businesses in the same way as they do to established big businesses.

C. Because they are growing and are seldom in equilibrium, new small businesses are not subject to the same applicability of formulas for cash flow and the ratio of debt to equity as established big businesses.

D. Because new small businesses are growing and are seldom in equilibrium, formulas for cash flow and the ratio of debt to equity do not apply to them in the same way as to established big businesses.

E. New small businesses are not subject to the applicability of formulas for cash flow and the ratio of debt to equity in the same way as established big businesses, because they are growing and are seldom in equilibrium.

1 代词指代:当有多个可以指代的对象时,选用代词容易造成混淆。

A do … , because B ….  如果用代词需要考虑指代是否指向A 合理

2平行结构:

formulas for cash flow and the ratio of debt to equity do not apply to them in the same way as formulas for cash flow and the ratio of debt to equity do to established big businesses. 在不引起歧义的前提下, in the same way as 后面可以省略主语和谓语。

+ 平行词 +(主谓)  ===》比较的主体是前后的宾语,

前后的主谓一致,主  省略·

new small businesses are not subject to the applicability of formulas for cash flow and the ratio of debt to equity in the same way as established big businesses

+ 平行词 +      ===》比较的主体是前后的主语,

前后的宾语一致,只能省略宾语。主语后有宾语部分,比较部分的主语后的系动词不能省,因为很容易造成后面的主语和前面的宾语比较的问题。

3.词法:

are not subject to the same applicability of  歧义而且不简洁。The same way as 合理。

To X the same as to Y, 伽利略人在船头的那道题。

4.逻辑意思清晰:先原因,后结果,表达的意思清晰明确

有三种结构:

1Because A , B .  2B, because A   3B because A

逗号很关键,没有逗号会引起歧义, because的优选结构是1)和2; 不用B because A(强调歧意)

例句

Young female ballet dancers and gymnasts sometimes fail to maintain good eating habits caused by the desire to be as thin as possible.

(A) Young female ballet dancers and gymnasts sometimes fail to maintain good eating habits caused by the desire to be as thin as possible. (分词词组修饰的是good eating habits, 明显错误. 显然 good eating habits are caused by the desire to be as thin as possible是不合逻辑的. Caused by 表示直接导致)

(B) Good eating habits sometimes fail to be maintained by young female ballet dancers and gymnasts caused by desiring to be as thin as possible.  choice A
(C) Because they desire to be as thin as possible, good eating habits are sometimes not maintained by young female ballet dancers and gymnasts.
(D) Because they desire to be as thin as possible, young female ballet dancers and gymnasts sometimes fail to maintain good eating habits.  
(E) Young female ballet dancers and gymnasts sometimes fail to maintain good eating habits because they desire to be as thin as possible.
先原因,后结果,表达的意思清晰明确

5.尤其是全划线的句子, 可能原来的语序本来就是大错特错嘛

 6.动词大于优选于名词??

 6.动词大于优选于名词??


[此贴子已经被作者于2007-8-8 16:14:14编辑过]

作者: elinaqu    时间: 2007-8-8 16:08

13.State officials report that soaring rates of liability insurance have risen to force cutbacks in the opera­tions of everything from local governments and school districts to day-care centers and recreational facilities.

A. rates of liability insurance have risen to force

B. rates of liability insurance are a force for

C. rates for liability insurance are forcing

D. rises in liability insurance rates are forcing

E. liability insurance rates have risen to force

1.  为什么have risen rises是多余的?   (重复问题-一级排除 A,D,E)

Soaring有高飞上升的意思,rise soaring 重复

2.  Rate of /for ?(词的用法  二级排除B

rate的意思是price charge 的时候不能接of, 应该接for payment for

rates  当表示费用时加介词 for

当表示比例 加介词 of

3. 为什么不用A force for而用are forcing(逻辑意思准确表达二级排除B)

是因为are a force for 不能够准确的表达the soaring rates 现在正在forcing cutbacks的动作。
动名词比名词更能表现出正在发生的动作状态。

4.动词优于名次(二级排除B)


[此贴子已经被作者于2007-8-8 16:10:01编辑过]

作者: elinaqu    时间: 2007-8-8 16:09

14. Paleontologists believe that fragments of a primate jawbone unearthed in Burma and estimated at 40 to 44 million years old provide evidence of a crucial step along the evolutionary path that led to human beings.

A. at 40 to 44 million years old provide evidence of

B. as being 40 to 44 million years old provides evidence of

C. that it is 40 to 44 million years old provides evidence of what was

D. to be 40 to 44 million years old provide evidence of

E. as 40 to 44 million years old provides evidence of what was

1.       主谓一致(单复数)(一级排除B,C,E

fragments 。。。。。。。。provide

2. 简洁:(一级排除 C. E )

what was … 多余

Sth is sth.

Sth does sth.

Sth is what is sth. (, 1. 不并列, 2. 用单个名词和what 从句的概念对称肯定不对)

例句

There is speculation that increasing cold weather was what may have been responsible for the Anasazi move from Mesa Verde to sites in other canyons.

(A) that increasing cold weather was what may have been

(B) whether increasing cold weather was what was

(C) that increasingly cold weather was what had been

(D) whether increasingly cold weather may have been what wasE

(E) that increasingly cold weather may have been

speculation that,,,,,名词Be what…..be 前后的东东要是一个东东啊

3.. Estimate to be / that/ at (词的用法     二级排除Aestimated to be ………  estimate 做被动,只能是 sth. be estimated to be.......Estimate that….. estimate 做主动,用estimate sth. that  B 中的estimated that it is      it 多余  

It is estimated that……

At+地点, at+点时间(点和线) 不能加年份的

金山词霸解释

Estimate  vt.  -mated, -mating估计,判断(常与that连用)估价,(常与at连用)估计

I estimate her age at 35.我估计她有35岁。

(与in连用)估计…到达


[此贴子已经被作者于2007-8-8 16:13:28编辑过]

作者: elinaqu    时间: 2007-8-8 16:16
以下是引用dennyhuang在2007-8-7 23:59:00的发言:

支持队友一下!加油,elina好认真啊。我第二遍过OG时也这样把每个考点写出来,没你这么细,但感觉对提高语法有很大帮助,相信你一定会掌握得更好。等我整理一下我的语法总结,也贴上来。

一起加油吧!!

谢谢,denny,一起努力啊


作者: 文明    时间: 2007-8-8 16:24
嗯,又是一个宏伟壮观建筑,楼主辛苦喽,支持支持
作者: elinaqu    时间: 2007-8-8 18:35

15. In his research paper, Dr. Frosh, medical director of the Payne Whitney Clinic, distinguishes mood swings, which may be violent without their being grounded in mental disease, from genuine manic-depressive psychosis.

A. mood swings, which may be violent without their being grounded in mental disease, from genuine manic-depressive psychosis

B. mood swings, perhaps violent without being grounded in mental disease, and genuine manic-depressive psychosis ,

C. between mood swings, which may be violent without being grounded in mental disease, and genuine manic-depressive psychosis

D. between mood swings, perhaps violent without being grounded in mental disease, from genuine manic-depressive psychosis

E. genuine manic-depressive psychosis and mood swings, which may be violent without being grounded in mental disease

1. distinguish 的用法from, between (考点就是distinguish,一级排除A,B,D,E

(常与from连用)使别于;有…特点
distinguish somebody/something from

a method of distinguishing cancer cells from normal tissue

(from的时候要小心,分辨两个的话优先用distinguish between  and)

distinguish between A and B . A/B 不同范畴进行区分。Between 不能省略,distinguish between A and B .which.  which 很清晰指向B.

distinguish A and B .which.  which 可能指向 A and 或者 B ,不清晰

Distinguish A from B  A B 的子集,从B中把A 挑出来。

此题是把假的病症和真的病症区分开, 不是把假的病症从真的病症里面区分出来)

2 情态动词要保持 may 不应该改成 perhaps.

3 代词:mood swings, which may be violent without their being grounded          Their 多余


[此贴子已经被作者于2007-8-8 18:41:12编辑过]

作者: elinaqu    时间: 2007-8-8 18:35

16. Unlike a typical automobile loan, which requires a fifteen- to twenty-percent down payment, the lease-loan buyer is not required to make an initial deposit on the new vehicle.

A. the lease-loan buyer is not required to make

B. with lease-loan buying there is no requirement of

C. lease-loan buyers are not required to make

D. for the lease-loan buyer there is no requirement of

E. a lease-loan does not require the buyer to make

1 比较结构平行(一级排除A,B,C,D

Unlike/ like A , B …..  A /B 要对等。(数的一致,逻辑意思的一致, 时态上的平行)比较时逻辑意思优先!

choice E is the only option that supplies an active verb form, does not require, to parallel requires

经典例句

经典例句

Unlike most warbler species, the male and female blue-winged warbler are very difficult to tell apart.

(A) Unlike most warbler species, the male and female blue-winged warbler are very difficult to tell apart.

(B) Unlike most warbler species, the gender of the blue-winged warbler is very difficult to distinguish.

(C) Unlike those in most warbler species, the male and female blue-winged warblers are very difficult to distinguish.

(D) It is very difficult, unlike in most warbler species, to tell the male and female blue-winged warbler apart.E

(E) Blue-winged warblers are unlike most species of warbler in that it is very difficult to tell the male and female apart.

感觉一下, 全划线和部分划线的不同---可以把全句重新放置!

比较对象要明确
   

a. most species ----------------------------the male and female blue-winged warbler是不能并列的
b. most warbler species------------------ the gender
c. those in most warbler species-------- the male and female blue-winged warblers
e. most species of warbler ---------------Blue-winged warblers

2.简洁

 动词比名词要好:is no requirement of / require


[此贴子已经被作者于2007-8-8 18:40:33编辑过]

作者: elinaqu    时间: 2007-8-9 09:49

17. Native American burial sites dating back 5,000 years indicate that the residents of Maine at that time were part of a widespread culture of Algonquian-speaking people.

A. were part of a widespread culture of Algonquian-speaking people

B. had been part of a widespread culture of people who were Algonquian-speaking

C. were people who were part of a widespread culture that was Algonquian-speaking

D. had been people who were part of a widespread culture that was Algonquian-speaking

E. were a people which had been part of a wide­spread, Algonquian-speaking culture

1.重复  (一级排除 C, D, E

people redundantly describes the residents rather than the larger group to which the residents belonged.

2    时态:( 一级排除 B, D )
过去完成时没有明确标志

3   修饰(逻辑主语不一致)(二级排除C,D,E )

culture that was Algonquian-speaking / Algonquian-speaking culture 修饰不正确

4. 代词:which 不能用来指代人….

5. 简洁定语从句 that be /who be /which be能省就省。


[此贴子已经被作者于2007-8-9 9:53:31编辑过]

作者: elinaqu    时间: 2007-8-9 09:50

18. Each of Hemingway's wives--Hadley Richardson. Pauline Pfeiffer. Martha Gelhom. and Mary Welsh --were strong and interesting women, very different from the often pallid women who populate his novels.

A. Each of Hemingway's wives--Hadley Richardson, Pauline Pfeiffer, Martha Gelhom, and Mary Welsh--were strong and interest­ing women,

B. Hadley Richardson, Pauline Pfeiffer, Martha Gelhorn, and Mary Welsh--each of them Hemingway's wives--were strong and, interesting women,

C. Hemingway's wives--Hadley Richardson, Pauline Pfeiffer, Martha Gelhom, and Mary Welsh--were all strong and interesting women,

D. Strong and interesting women—Hadley Richardson, Pauline Pfeiffer, Martha Gelhom, and Mary Welsh--each a wife of Hemingway, was

E. Strong and interesting women—Hadley Richardson, Pauline Pfeiffer, Martha Gelhom, and Mary Welsh--every one of Hemingway's wives were

1 主谓一致,单复数(一级排除)each of … + 谓语动词单数(排除A
代词 ==each of them Hemingway's wives- 数不一致an illogical statement because each cannot be wives; each can be one of the wives, or a wife ( 排除 B ) womenwas ( 排除 D )every one in E  does not agree with the plural verb were (排除E )

2.     简洁代词太多造成指代不明

3. 句子重心的保持. è Strong and interesting women 改变了原句的主语


[此贴子已经被作者于2007-8-9 9:54:38编辑过]

作者: crowkiller    时间: 2007-8-9 09:56
以下是引用elinaqu在2007-8-9 9:49:00的发言:

17. Native American burial sites dating back 5,000 years indicate that the residents of Maine at that time were part of a widespread culture of Algonquian-speaking people.

A. were part of a widespread culture of Algonquian-speaking people

B. had been part of a widespread culture of people who were Algonquian-speaking

C. were people who were part of a widespread culture that was Algonquian-speaking

D. had been people who were part of a widespread culture that was Algonquian-speaking

E. were a people which had been part of a wide­spread, Algonquian-speaking culture

1.重复  (一级排除 C, D, E

people redundantly describes the residents rather than the larger group to which the residents belonged.

2    时态:( 一级排除 B, D )
过去完成时没有明确标志       ====》 这一点在PrepSC里面有不一致的地方,要注意。

3   修饰(逻辑主语不一致)(二级排除C,D,E )

culture that was Algonquian-speaking / Algonquian-speaking culture 修饰不正确

4. 代词:which 不能用来指代人….

5. 简洁定语从句 that be /who be /which be能省就省。



作者: elinaqu    时间: 2007-8-9 10:32

2    时态:( 一级排除 B, D )
过去完成时没有明确标志       ====》 这一点在PrepSC里面有不一致的地方,要注意。

那我可不可以理解时态不可以作为一级排除,不是最优先的呢?在三板斧里,也有说道----。我再多说一点-“时态”和“代词”的问题。这是ETS的最后防线,也是整个英语的难点。当看到这些情况的时候,如果我们没有绝对的把握,请不要主观的抛弃你认为不正确的选项,而应该细致的比较两个选项其他的不同点,在做题的过程中,我们可能会碰到比较难的语法点,一定要本着“避实就虚”的原则,不要想一口吃个大胖子,一次就扔掉三个选项,请指教,谢谢killer


作者: crowkiller    时间: 2007-8-9 10:45
以下是引用elinaqu在2007-8-9 10:32:00的发言:

2    时态:( 一级排除 B, D )
过去完成时没有明确标志       ====》 这一点在PrepSC里面有不一致的地方,要注意。

那我可不可以理解时态不可以作为一级排除,不是最优先的呢?在三板斧里,也有说道----。我再多说一点-“时态”和“代词”的问题。这是ETS的最后防线,也是整个英语的难点。当看到这些情况的时候,如果我们没有绝对的把握,请不要主观的抛弃你认为不正确的选项,而应该细致的比较两个选项其他的不同点,在做题的过程中,我们可能会碰到比较难的语法点,一定要本着“避实就虚”的原则,不要想一口吃个大胖子,一次就扔掉三个选项,请指教,谢谢killer

对,等你做到PrepSC阶段,你会发现过去完成时/现代完成时都有没有明确对应的对象而出现的。所以我同意上面三板斧的看法,时态和代词不要放在前面排除。先找其他的,到最后剩下2-3个选项时再考虑时态。而且用时态排除也要尽量找有非常明确的标志的: 比如说 since + 时间 就是现在完成时的标志。但是没有这个标志不见得用完成时态就错误。
作者: elinaqu    时间: 2007-8-9 10:56

怎么感觉,似乎只有相对的绝对和相对的相对啊。


作者: crowkiller    时间: 2007-8-9 11:15
以下是引用elinaqu在2007-8-9 10:56:00的发言:

怎么感觉,似乎只有相对的绝对和相对的相对啊。

对,体会到这一点就开始有排除的感觉了。

还是有一些最重要的原则的,主要是句子的大逻辑结构完整性上: 比如说 Run - 0n ,比如说句子缺主语,谓语。

这些说起来很容易,但是GMAC会非常狡猾的往句子中插入一大堆什么从句啊,插入语啊来混淆。

练到很快能抓住大结构,而不管其他的干扰成分时。有很多大家都认为很难的所谓长划线的SC题目就迎刃而解了。

一边做一边好好体会那个三板斧吧。

PS:OG10SC 19题是最好的比较结构题目,好好总结一下。


作者: elinaqu    时间: 2007-8-9 18:09

这一贴全是摘录killer总结的宝贵资料

19. In addition to having more protein -than wheat does, the protein in rice is higher quality than that in wheat, with more of the amino acids essential to the human diet.

A. the protein in rice is higher quality than that in

B. rice has protein of higher quality than that in

C. the protein in rice is higher in quality than it is in

D. rice protein is higher in quality than it is in

E. rice has a protein higher in quality than

1.      修饰 (一级排除  A,C,D )

前置定语修饰离得最近的主语,主语要和前置定语的逻辑关系一致

2. 比较结构 . 比较结构前后核心要一致。( 二级排除E )Choice E needs either that in or does after wheat to make a complete and logical comparison.

 对主系表的句型主语比较:A is/has + adj. than B …. AB为对等名词, 此时不用补出系动词。

 对主谓宾全的句型主语比较:A do sth than B do.AB为对等名词,加do是为了显示这是一个可以发出动作的名词,因而确定是主语,进而确定是主语对比)

介宾比较 A do sth of C1 by D1 than of C2 by D2.C1C2D1D2对等名词)

宾语比较 A do C1 than C2.(C1C2为对等名词)

状语比较A do sth than usual/everbefore/people expected(直接加状语)

ETS就这几下子,原则就是清楚地指示什么跟什么比。按照这个原理,对下一种句型只有主谓,没有宾语的句子,因为没有宾语,只可能有主语比较,因而不需要重复do来确认这是一个可发出动作的主语,

所以其比较形式如下:
I eat faster than you.注意与下句区别 I eat apple faster than you do 如果写成 I eat apple faster than you ETS的理解,从语法上就变成我吃苹果比吃你快,成了宾语比较。逻辑上显然不通。

Rice has protein of higher quality than wheat does.(主语比较,Rice wheat比)意义:米有高质蛋白质,麦没有。
Rice has protein of higher quality than of lower quality(介宾比较,of higher of lower比)意义:米有高质蛋白质而非低质蛋白质
Rice has protein of higher quality than that in wheat(宾语比较,protein protein(that指代) 意义:米中的蛋白质比麦中蛋白质质量高
Rice has protein of higher quality than sugur.(宾语比较,protein sugar比)意义:米中的高质蛋白质比糖多。
Rice has protein of higher quality than usual/everbfore/1000 years ago/people expected(状语比较)意义:米比通常/经往/多年前/人们预测/的蛋白质质量要高。

AS族的比较大同小异,as 可单独引导比较,也可以复合成as many as, as hard as 等形式,但ETS宗旨不变,

Sand road costs twice as many to build as to manintain(类似于介宾比较)
    
Sand road costs twice as many to build as stone road do(主语比较,

Sand road costs the govement twice as many to build as the residents.(宾语比较)

 ETS考过AS比较的习惯用法(不受上述限制,如下) AS poor as they are, they can not afford a car. They have 800million students, as many as have enrolled in our school. (注意以上都是用逗号隔开的句子,AS在这里的作用更类似连词,连接一个主句和一个从句)

注意AS有时不表比较的如as you word hard, you will pass the exam.表原因了。
There be 句型等同于一般的主谓宾俱全的句型 OG89

There is one PC for every 32 pupils in American four time as many as there were four years ago(主语比较)

There is one PC for every 32 pupils in American four time as many as DVD player(宾语比较)

There is one PC for every 32 pupils in American four time as many as for school teachers.(介宾比交)

There is one PC for every 32 pupils in American four time as many as in china(介宾比较)

3 代词the protein in rice is higher in quality than it is in / rice protein is higher in quality than it is in  è it 指代 the protein in rice / rice protein 和后面的 in wheat ,逻辑意思不合理


[此贴子已经被作者于2007-8-9 18:21:02编辑过]

作者: elinaqu    时间: 2007-8-9 18:09

20. An array of tax incentives has led to a boom in the construction of new office buildings; so abundant has capital been for commercial real estate that investors regularly scour the country for areas in which to build.

A. so abundant has capital been for commercial real estate that

B. capital has been so abundant for commercial real estate, so that

C. the abundance of capital for commercial real estate has been such,

D. such has the abundance of capital been for commercial real estate that

E. such has been an abundance of capital for commercial real estate,

1.       惯用语(一级排除B,C,D,Eso/such  … that 结构

such … that  such 表示这种,无强调; so .. that 有强调的含义

A. so + adj. , so that 不合理

B.倒装:

根据主语和谓语相对位置,当句子的全部谓语都位于主语之前称之为完全倒装,当句子的部分谓语位于主语之前称为局部倒装句。举例:
1.Here comes the bus!(完全倒装句)

2.Only in this way can a good result be achieved.(局部倒装句)
本题A选项为局部倒装句,当构成结果状语从句so---thatso位于句首表示强调。

so..that (正常)capital has been so abundant that investors scour the country for areas .. 倒裝:so abundant has capital been that investors scour the country for areas ..→是部份倒裝型式
such..that 正常:an abundance of capital has been such that investors scour the country for areas.. 倒裝:such has been an abundance of capital that investors scour the country for areas..←完全倒裝型式 

C.Such/ so that 比较:

1.首先一点要明确, such 是形容词, 修饰名词或名词性的结构(such a good job, such good work, such a good person,...), so 是副词, 修饰形容词 ( a so good job, so good work, a so good person...)1. "such functions to mean "of a kind" rather than to intensify abundant." - 这样说是很有根据的. 因为such加强的是后面的名词, 重点是事物上. so加的强是后面的形容词. 这里的需要语感去体会.

2. 我个人在改错题中的一个发现是, 如果选项中有到装句, 要特别留意. "so abundant has capital been for commercial real estate that", 就是. 到装句成为正确答案的可能性非常大. , (大家去验证一下, 有例外的请发贴). 原因可能是因为到装句是英语中一种固定的结构, 它可以突出,强调重点, 使表达更清晰, GMAT对句子的要求一致. 而到装句结构和一般的句子不一样, 容易被人看成是错句!

2. 单复数:A of B + 谓语,谓语的数和 A一致。An array of tax incentives has led to


[此贴子已经被作者于2007-8-9 18:21:45编辑过]

作者: elinaqu    时间: 2007-8-9 18:10

21. Defense attorneys have occasionally argued that their clients' misconduct stemmed from a reaction to something ingested, but in attributing criminal or delinquent behavior to some food allergy, the perpetrators are in effect told that they are not responsible for their actions.

A. in attributing criminal or delinquent behavior to some food allergy

B. if criminal or delinquent behavior is attributed to an allergy to some food

C. in attributing behavior that is criminal or delinquent to an allergy to some food

D. if some food allergy is attributed as the cause of criminal or delinquent behavior

E. in attributing a food allergy as the cause of criminal or delinquent behavior

1. 修饰(一级排除 A,C,,E

前置定语的中心词指向句子主语,是否合适。A, C, and E, in attributing ... behavior modifies the perpetrators, producing the illogical statement that the perpetrators rather than the defense attorneys are attributing behavior to food allergies.(动词attributing的发起者!!!!!)

That be /which be + 形容词都省略that be /which be (排除C

2 习语(二级排除 D )

In the correct form of the expression, one attributes x, an effect, to y, a cause; è将一个现象归咎于某个东东x is attributed to y == x 归因于y , or, if a passive construction is used, x is attributed to y. ,,,,x is attributed to y == x 归因于y


[此贴子已经被作者于2007-8-9 18:18:03编辑过]

作者: crowkiller    时间: 2007-8-9 19:53

体会下这个题目:

http://forum.chasedream.com/dispbbs.asp?boardid=23&id=267884&star=1#267884


作者: elinaqu    时间: 2007-8-10 09:58

22. The voluminous personal papers of Thomas Alva Edison reveal that his inventions typically sprang to life not in a flash of inspiration but evolved slowly from previous works.

A. sprang to life not in a flash of inspiration but evolved slowly

B. sprang to life not in a flash of inspiration but were slowly evolved

C. did not spring to life in a flash of inspiration but evolved slowly

D. did not spring to life in a flash of inspiration but had slowly evolved

E. did not spring to life in a flash of inspiration but they were slowly evolved

 

1.  并列结构  (一级排除
                A,B,D

not … but 结构
            

            
前后的形式,时态都要绝对平行

并列结构的时态要一致才是并列!!!è互相并列的成分是不能作为时间参照点的

 

1.       被动语态(二级排除 E )

被动语态在GMAT中必须有理由才能出现。(使用被动态一定要由根据)è出现by

 


作者: elinaqu    时间: 2007-8-10 09:58

23. A Labor Department study states that the numbers of women employed outside the home grew by more than a thirty-five percent increase in the past decade and accounted for more than sixty-two percent of the total growth in the civilian work force.

A. numbers of women employed outside the home grew by more than a thirty-five percent increase

B. numbers of women employed outside the home grew more than thirty-five percent

C. numbers of women employed outside the home were raised by more than thirty-five percent

D. number of women employed outside the home increased by more than thirty-five percent

E. number of women employed outside the home was raised by more than a thirty-five percent increase

 

1.       简洁(一级排除
                A,E
grew/raise  increase 重复

the phrase grew by more than a thirty-five percent increase is redundant and wordy,

was raised by ... increase is redun­dant.

2.       词法
            
(二级排除 A,B,C )

表示统计数据用
            
单数 the number , the number of ……..

1.       被动语态(二级排除 C,E )
                

被动语态要有合理的发起

because there is no identifiable agent responsible for the raising of the number of women employed.用被动要有动作的逻辑发起


作者: elinaqu    时间: 2007-8-10 09:59

24. The first decision for most tenants living in a building undergoing being converted to cooperative ownership is if to sign a no-buy pledge with the other tenants.

A. being converted to cooperative ownership is if to sign

B. being converted to cooperative ownership is whether they should be signing

C. being converted to cooperative ownership is whether or not they sign

D. conversion to cooperative ownership is if to sign

E. conversion to cooperative ownership is whether to sign

 

1, 简洁(一级排除
                A,B,C

the phrase being converted is awkward and redundant, since the sense of process indicated by being has already been conveyed by undergoing.

 

2. 词法(二级排除 D )

 表示是否用 whether, 表示条件用if

 及物动词+名词的形式
            
+分词的形式更加合理。uses the noun conversion, which grammatically completes the phrase begun by undergoing. undergo及物动词. 不能接分词.

 

3. 并列结构
                

A is B,  A/B 平行。 ==
            
不定式可以作为名词性结构和名词平行。

The first decision 。。。 is whether to …..

to sign, that functions as a noun equivalent of decision

并列处处有!!!!系表结构是并列!!!!!!!!!!!!!!.

例句
                

140. It may be another fifteen years before spacecraft from Earth again venture to Mars, a planet now known to be cold, dry, and probably lifeless.
(A) again venture to Mars, a planet now known to be
(B) venture to Mars again, a planet now known for being
(C) will venture to Mars again, a planet now known as being
(D) venture again to Mars, a planet that is known now to be
(E) will again venture to Mars, a planet known now as being

 

C,E的将来时态不对, 排除

Bfor being不对: for要么是目的关系, 要么是因果关系
for being
的意思是因为MARSblablabla, 偶们才认识这个星球.---有点离谱

只有to be才是a planet的等价物!!!是名词性质!!!
                

里面的do又是动词
            
绝对比动名词being/doing好百倍

For being, as being都是介词短语, 不是名词性质的等价物!!!

Dthat is多余, 不对

 

4undergo 的用法v 经历[transitive not in passive]

if you undergo a change, an unpleasant experience etc, it happens to you or is done to you

The country has undergone massive changes recently.

He has been released from prison to undergo medical treatment in the United States.


作者: elinaqu    时间: 2007-8-10 09:59

25. The end of the eighteenth century saw the emergence of prize-stock breeding, with individual bulls and cows receiving awards, fetching unprecedented prices, and excited enormous interest whenever they were put on show.

A. excited

B. it excited

C. exciting

D. would excite

E. it had excited

 

1.       平行结构(一级排除
                A,B,D, E

 A , B , and C 平行
            
,动名词,动名词,and动名词,

可不可以A  a  and b, and B ? 可不可以作为一个排除标准

 

2.  代词指代(二级排除 B, E )

  it 无指代对象


作者: elinaqu    时间: 2007-8-10 09:59

26. Of all the possible disasters that threaten American agriculture, the possibility of an adverse change in climate is maybe the more difficult for analysis.

A. is maybe the more difficult for analysis

B. is probably the most difficult to analyze

C. is maybe the most difficult for analysis

D. is probably the more difficult to analyze

E. is, it may be, the analysis that is most difficult

 

1. 绝对错误比较结构:(一级排除
                A,D

 表示比较对象一个是另外一个的全子集的时候,用最高级比较形式

1.  简洁(一级排除  E )

when inserted into the sentence, produces an illogical structure: the possibility ... is... the analysis that.

2.  平行结构一级排除
                C

A is B  B 可以为不定式和
            
名词A对应。

difficult should be completed by the infinitive to analyze.

an infinitive,  to analyze, functions as a noun equivalent

可以说: To analyze is difficult. 但是不能说For analysis is difficult.

 

1.  词法(二级排除 C )

probably / maybe probably 对应概率, maybe表示能够。         

(maybe是个口语表达,书面语应该用probably)=è其实这个说法好牵强

真正的理由是: possibility表示概率, 那么就要用probably, 表示概率的词语, 来对应

maybe/can/could都是能够, 不是概率


作者: elinaqu    时间: 2007-8-10 10:00

27. Published in Harlem, the owner and editor of the Messenger were two young journalists. Chandler Owen j and A. Philip Randolph, who would later make his reputation as a labor leader.

A. Published in Harlem, the owner and editor of the Messenger were two young journalists. Chandler Owen and A. Philip Randolph, who would later make his reputation as a labor leader.

B. Published in Harlem, two young journalists, Chandler Owen and A. Philip Randolph, who would later make his reputation as a labor leader, were the owner and editor of the Messenger.

C. Published in Harlem, the Messenger was owned and edited by two young journalists, A. Philip Randolph, who would later make his reputation as a labor leader, and Chandler Owen.

D. The Messenger was owned and edited by two young journalists, Chandler Owen and A. Philip Randolph, who would later make his reputation as a labor leader, and published in Harlem.

E. The owner and editor being two young journalists, Chandler Owen and A. Philip Randolph, who would later make his reputation as a labor leader, the Messenger was published in Harlem.

 

1.修饰
            

            
主语(一级排除
                A, B

A. 前置定语的中心词指向句子的主语。

 

2. 逻辑意思明确(二级排除D, E
                

并列结构的非限制性定语从句的修饰。
                

A and B , who/which….  从句修饰 A and B;A,who/which …. . and B.从句只修饰 A

c makes it clear that the clause beginning who refers to Randolph. A, B, D, E里面的who would later make his reputation修饰有问题, 形式上修饰复合短语, Chandler Owen and A. Philip Randolph, 但是两个人于his又不对应

3.修饰的有效性

修饰结构离修饰语太远不合理

the phrase and published in Harlem is too remote from the Messenger to modify it effectively (注意,也算是一种修饰错误).

 

4.  简洁:(二级排除E

 A being B , being 不简洁。

 

5.. 并列结构:
            
注意逻辑意思上的平行。The owner and editor, the Messenger 是同位语,但是实际意思有问题,排除E


作者: elinaqu    时间: 2007-8-10 10:00

28. The rise in the Commerce Department's index of leading economic indicators suggest that the economy should continue its expansion into the coming months, but that the mixed performance of the index's indi­vidual components indicates that economic growth will proceed at a more moderate pace than in the first quarter of this year.

A. suggest that the economy should continue its expansion into the coming months, but that

B. suggest that the economy is to continue expansion in the coming months, but

C. suggests that the economy will continue its expanding in the coming months, but that

D. suggests that the economy is continuing to expand into the coming months, but that

E. suggests that the economy will continue to expand in the coming months, but

 

1.句子完整性一级排除
                A, C, D—but that

the that appearing after but creates a subordinate clause where an independent clause is needed for the new subject, mixed performance

 

2.  主谓一致(二级排除 B)
                

 the rise … suggests

 

3. 并列( 大并列& 小并列)  (二级排除 B )

The rise… ofsuggests thatwill continuein the coming months, but the mixed performance of indicates thatwill proceedin the first quarter of this year.

 

4. 词法:
            
表示即将来到的几个月: in the coming month 不能用into

 

5. 时态:
            
表示将来发生的最好用将来时态

That this tense is called for is indicated both by the future time to which the coming months refers and by the parallel verb form will proceed in the nonunderlined part of the sentence (并列句子, 开头不敢用的).

 

6. V.> N
                

表示动作,动词比动名词优先。


作者: elinaqu    时间: 2007-8-10 10:01

29. In three centuries--from 1050 to 1350--several million tons of stone were quarried in France
                for the building of eighty cathedrals, five hundred large churches, and some tens of thousands of parish churches.

A. for the building of eighty cathedrals, five hundred large churches, and some

B. in order that they might build eighty cathedrals, five hundred large churches, and some

C. so as they might build eighty cathedrals, five hundred large churches, and some

D. so that there could be built eighty cathedrals, five hundred large churches, and

E. such that they could build eighty cathedrals, five hundred large churches, and

 

1代词:(一级排除
                B,C,E

they 没有指代对象

2. 简洁:(二级排除 D)

there could be 不简洁

3. 表示目的的结构:(二级排除 D)

for + n. 介宾结构
            
,不定式

 

so that 对不对?


作者: elinaqu    时间: 2007-8-10 10:01

30. What was as remarkable as the development of the compact disc has been the use of the new technology to revitalize, in better sound than was ever before possible, some of the classic recorded performances of the pre-LP era.

A. What was as remarkable as the development of the compact disc

B. The thing that was as remarkable as developing the compact disc

C. No less remarkable than the development of the compact disc

D. Developing the compact disc has been none the less remarkable than

E. Development of the compact disc has been no less remarkable as

 

1.绝对错误习语(一级排除 D, E )

 no less ….than  不能用 no less .. as 或者 none the less … than

 

1.       平行结构(一级排除 A, B-时态
                )

时态一致,用名词对名词,时态的一致性。插入语的时态可以不一致,此处表示和过去比较。

cause inconsistencies in verb tense: the use of the new technology cannot logically be described by both the present perfect has been and the past was.关注系表结构, 系表结构等于是并列结构, 时态要一致, 概念要对等

 

系表结构的平行  A is/has been B A/B 要平行对等
            

            
逻辑意思上的对等。。。

developing the compact disc is not parallel to the use of new technology to revitalize ... performances

 

What was in A and The thing that was in B 为什么wordy ,什么时候应该省

 

中间就可以用过去时态, 因为是插入语表示的是和过去的比较, 当然要was  - ???


作者: elinaqu    时间: 2007-8-10 10:01

31. Unlike computer skills or other technical skills, there is a disinclination on the part of many people to recognize the degree to which their analytical skills are weak.

(A)  Unlike computer skills or other technical skills, there is a disinclination on the part of many people to recognize the degree to which their analytical skills are weak.

(B)  Unlike computer skills or other technical skills, which they admit they lack, many people are disinclined to recognize that their analytical skills are weak.

(C)  Unlike computer skills or other technical skills, analytical skills bring out a disinclination in many people to recognize that they are weak to a degree.

(D)  Many people, willing to admit that they lack computer skills or other technical skills, are disinclined to recognize that their analytical skills are weak.

(E)   Many people have a disinclination to recognize the weakness of their analytical skills while willing to admit their lack of computer skills or other technical skills.

 

1比较结构(一级排除 A, B )

Unlike A , B A/B 要同类对等

 

2.简洁(二级排除 C, E )

it is awkward and unidiomatic to say skills bring out a disinclination.
                

     动词 > 名词

be disinclined a disinclination 要好

 

3.代词指代不清

they is unclear, and weak to a degree changes the meaning of the original statement.

 

1.  词法:

admit their lack should be
                    admit to their lack.

 admit

admit that... 承认 ; admit sb. to do sth. 允许 ; admit to doing sth.  不能是 admit sth.

While 的用法:

1while
            
...
            
的意思,引导时间状语从句;主语相同,谓语为be可同时省略
            
;主语相同,谓语为一般动词,则动词变为分词
            

                

形式:主句,while+ -ing/a.

2
                 while

            
而,然而的意思,表对比转折;主语谓语动词不管怎样都不能省,应该完整写出while they are willing........


作者: elinaqu    时间: 2007-8-10 10:02

32. Some buildings that were destroyed and heavily dam­aged in the earthquake last year were constructed in violation of the city's building code.

(A)  Some buildings that were destroyed and heavily damaged in the earthquake last year were

(B)  Some buildings that were destroyed or heavily damaged in the earthquake last year had been

(C)  Some buildings that the earthquake destroyed and heavily damaged last year have been

(D)  Last year the earthquake destroyed or heavily damaged some buildings that have been

(E)   Last year some of the buildings that were destroyed or heavily damaged in the earthquake had been

 

1.逻辑意思的合理性(一级排除 A, C )

 and/or 的选择,destroy/damage 不能同时发生。

逻辑意思决定了时态 ==
                construct
发生在 destroy 之前
                

时态的选择决定于
            
从句和主句的动作发生的先后。(排除 D )

Choices C and D use the present perfect tense incorrectly, saying in effect that the buildings have been constructed after they were destroyed last year.

 

2. 改变句子中心(二级排除 E )

 时态选择与时间状语:
                

根据主从句的逻辑关系决定动作发生的先后,决定时态。

一般过去式的时间状语用时间点,如此题中的 last year. 完成时态用时间段做时间状语,通常用 since ….

时间状语的改变位置从从句到主句
            
会造成句子意思的改变,需要注意。

 --从句变成主句
            
或者
            
主句变成从句
            
都会改变句子的中心意思  选项D

 --时间状语的改变位置从从句到主句
            
会造成句子意思的改变,  选项E

Choice E suggests that the construction of the buildings, rather than the earthquake, occurred last year, thus making the sequence of events unclear.

句子中心不能改变
            
è D 改变了从句成主句, E 的时间状语改变位置造成了句子意思改变。


作者: elinaqu    时间: 2007-8-10 10:02

33. From the earliest days of the tribe, kinship determined the way in which the Ojibwa society organized its labor, provided access to its resources, and defined rights and obligations involved in the distribution and consumption of those resources.

(A)  and defined rights and obligations involved in the distribution and consumption of those resources

(B)  defining rights and obligations involved in their distribution and consumption

(C)  and defined rights and obligations as they were involved in its distribution and consumption

(D)  whose rights and obligations were defined in their distribution and consumption

(E)   the distribution and consumption of them defined by rights and obligations

 

1平行结构:(一级排除 B,D,E )

 谓语过去式平行结构
            
A+ed, B+ed, and C+ed

 

2.代词: (二级排除 C )

its has no logical referent

  补出概括性名词消除不清楚的代词是正确选项。

 

3. 修饰:n. whose +。。。
            
需要考察 whose 指向的名词是否合理

4. 逻辑意思  defined by rights and obligations 改变了逻辑结构的主语

5. 简洁 n. + 分词
            
n. + 定语从句
            
that/as + be + v.+ed 要好

对否定被动的倾向性?


作者: elinaqu    时间: 2007-8-10 10:02

34. A report by the American
                
Academy
for the Advance­ment of Science has concluded that much of the currently uncontrolled dioxins to which North Ameri­cans are exposed comes from the incineration of wastes.

(A)  much of the currently uncontrolled dioxins to which North Americans are exposed comes

(B)  much of the currently uncontrolled dioxins that North Americans are exposed to come

(C)  much of the dioxins that are currently uncon­trolled and that North Americans are exposed to comes

(D)  many of the dioxins that are currently uncontrolled and North Americans are exposed to come

(E)   many of the currently uncontrolled dioxins to which North Americans are exposed come

 

 

1单复数一致:(一级排除 A,B,C )

dioxins 为可数复数名词应用many, come

 主谓单复数一致:

 修饰词的单复数一致: much + 不可数名词
            
many + 可数名词

2简洁/平行结构:二级排除 D )

n. + that/ whose /which be + adj. 没有 adj. + n. 简洁。

currently uncontrolled dioxins / dioxins that are currently uncontrolled

D requires that before North Americans, to be grammatically complete.

dioxins to which North Americans are exposed / many of the dioxins ……  and North Americans are exposed to 改变了句子意思和逻辑关系。

 

3
            
修饰:
            
修饰顺序: a report by … for ……

4.平行结构
            
逻辑意思决定了分句/主句的平行。改变原句的层次会改变句子意思。


作者: caicaichang    时间: 2007-8-10 11:13
elinaqu!加油哦我相信你可以坚持下来的!
作者: elinaqu    时间: 2007-8-10 13:30

35. In June of 1987, The Bridge of Trinquetaille, Vincent van Gogh's view of an iron bridge over the Rhone sold for $20.2 million and it was the second highest price ever paid for a painting at auction.

A. Rhone sold for $20.2 million and it was

B. Rhone, which sold for $20.2 million, was

C. Rhone, was sold for $20.2 million,

D. Rhone was sold for $20.2 million, being

E. Rhone, sold for $20.2 million, and was

 

1.       句子的完整性(一级排除A, D )

AB+ V. 结构中 B 后面的逗号不能省略,B作同位语

without the comma, the phrase appears to be part of the main clause, and it is thus unclear what noun should govern the verb sold

2.       逻辑意思
            
二级排除B, E )

平行结构:
            
系表结构的意思平行:  The Bridge of Trinquetaill was the second price 意思不一致

3.  代词it in A has no logical referent

4. 修饰
            
非限制性定语从句 which 指向词需要考察

5. being 一般错

类似题型例句

56. Architects and stonemasons, huge palace and temple clusters were built by the Maya without benefit of the wheel or animal transport.

(A) huge palace and temple clusters were built by the Maya without benefit of the wheel or animal transport

(B) without the benefits of animal transport or the wheel, huge palace and temple clusters were built by the Maya

(C) the Maya built huge palace and temple clusters without the benefit of animal transport or the wheel

(D) there were built, without the benefit of the wheel or animal transport, huge palace and temple clusters by the MayaC
                

(E) were the Maya who, without the benefit of the wheel or animal transport, built huge palace and temple clusters

这道题就是典型的“n1, n2+v".

综合这道题和stoneren 的解释与举例,我想是不是这样:

n1是同位补充解释时,n2后不加逗号是可以的。比如这个og56

n2充当同位补充解释时,就需要加逗号。比如og35

同位语虽然是等价的意思, peajack, 我认为主语和同位语的位置不能互换.

如果architects and stonemasons作主语, 那在逻辑上是说不通的. 因为是the Maya建造了那些宫殿和庙宇without of benefit..., 而不是随便的一些建筑师和石头匠(泛指).

同位语是解释说明主语,把凡高的view定义成The Bridge of T不合逻辑;只有把The Bridge of T解释成凡高的画才行。

可以参考 OG 220

220.
                
For almost a hundred years after having its beginning in 1788,
England exiled some 160,000 criminals to Australia.

(A) For almost a hundred years after having its beginning in 1788,

(B) Beginning in 1788 for a period of a hundred years,

(C) Beginning a period of almost a hundred years, in 1788,

(D) During a hundred years, a period beginning in 1788,E

(E) Over a period of a hundred years beginning in 1788,Aside from being wordy and awkward, choice A is illogical: because its refers grammatically to England, A states nonsensically that England had its beginning in 1788. Choice B is similarly illogical, because the initial verb phrase Beginning in 1788... modifies England, the subject of the main clause. Choice C is imprecise, saying that England in 1788 was Beginning a period... but not conveying the sense that anything happened within that period. Choice D is awkward and unidiomatic, and nonsensically suggests that a hundred years is defined as a period beginning in 1788. Precise and idiomatically phrased, choice E is best.

 


作者: elinaqu    时间: 2007-8-10 13:32

36. Bufo marinus toads, fierce predators that will eat frogs, lizards, and even small birds, are native to South America but were introduced into Florida during the 1930's in an attempt to control pests in the state's vast sugarcane fields.

A. are native to South America but were introduced into Florida during the 1930's in an attempt to control

B. are native in South America but were introduced into Florida during the 1930's as attempts to control

C. are natives of South America but were introduced into Florida during the 1930's in an attempt at controlling

D. had been native to South America but were introduced to Florida during the 1930's as an attempt at controlling

E. had been natives of South America but were introduced to Florida during the 1930's as attempts at controlling

 

1
            
时态一级排除DE )

 表示事实和持续存在的
            

            
一般现在式
            

            
过去完成式
            
有表示过去存在但是过去已经结束的状态In D and E, had been inaccurately implies that the toads are no longer native, or indigenous, to South America,

2.句子逻辑意思二级排除B )

Both as attempts in B and E and as an attempt in D are wrong because the attempt consists not of the toads themselves, but of their introduction into the environment. The correct phrase, in an attempt, should be completed by an infinitive (here, to control), as in A.(重要,体会如何用词,蟾蜍自己本身不是一个attempt) as sth  表示作为, 那么作为一定有个东东是作为的主语

as attempts / in an attempt  A as B , A B 对等
            
此处 toads 不可能和 attempts 对等。
                In attempt to
在。。。这样的一个动作中。

 

3. 表示目的三级排除C )

表示目的用不定式比较好

4.  词法 

A. were introduced into Floridaè表示介绍到……地方
            
introduced to Florida is unidiomatic. è表示介绍给知道

B. native to / native of / native in  to 合理,

C. as attempts / in an attempt  A as B , A B 对等
            
此处 toads 不可能和 attempts 对等。
                In attempt to
在。。。这样的一个动作中。

 


作者: elinaqu    时间: 2007-8-10 13:32

37. While some academicians believe that business ethics should be integrated into every business course, others say that students will take ethics seriously only if it would be taught as a separately required course.

A. only if it would be taught as a separately required course

B. only if it is taught as a separate, required course

C. if it is taught only as a course required separately

D. if it was taught only as a separate and required course

E. if it would only be taught as a required course, separately

 

1.句子逻辑意思一级排除CD, E )

Only if 不能变成 If + .. + Only  强调的意思不一样

only in C, D, and E should immediately precede the entire clause that it is meant to modify.

Only if只有才

2. 虚拟语气(二级排除A )

in sentences expressing a conditional result (x will happen if y happens), the verb of the main clause should be in the future tense and the verb of the if clause should be in the present indicative. x will happen if y happens  条件状语从句的时态一定要是主句的一级过去式。
            

            
一般将来时 if 一般现在时
            

            
一般现在时 if 一般过去式;

3  修饰:

Also, the intended meaning is distorted when the adverb separately is used to modify required, as in A and C, or taught, as in E;  B correctly uses the adjective separate to modify course.

体会所要表达的原始的意思(separately的修饰错误。)

adv. + adj.+ n . adv. 修饰 adj. 表示 adj. 发生的状态; adj. , adj. + n.  adj. 修饰 n.  ==
            
根据逻辑意思决定
            
是那种形式。


作者: elinaqu    时间: 2007-8-10 13:33

38. Scientists have observed large concentrations of heavy-metal deposits in the upper twenty centimeters of Baltic Sea sediments, which are consistent with the growth of industrial activity there.

A. Baltic Sea sediments, which are consistent with the growth of industrial activity there

B. Baltic Sea sediments, where the growth of industrial activity is consistent with these findings

C. Baltic Sea sediments, findings consistent with its growth of industrial activity

D. sediments from the Baltic Sea, findings consistent with the growth of industrial activity in the area

E. sediments from the Baltic Sea, consistent with the growth of industrial activity there

 

 

1. 修饰
            
(一级排除A,B,E )

非限制定语从句 which/ where 指向前面的名词,
            
考察

In A and B the words which and where appear to refer to sediments, and in E it is not clear what consistent describes

2.代词(二级排除C )

 补出抽象名词
            
代替代词
            
消除
            
歧义

 

3. 独立主格结构
            

            
独立主格
            
补出名词合理

虽然没说,但是这样的表达:sediments from the Baltic sea
            
要优于
            
Baltic sea sediments.
同样的表达有increase in sales, 等等,仔细体会这些细小的差异地方。


作者: elinaqu    时间: 2007-8-10 13:34

39. For members of the seventeenth-century Ashanti nation in Africa, animal-hide shields with wooden frames were essential items of military equipment, a method to protect warriors against enemy arrows and spears.

A. a method to protect

B. as a method protecting

C. protecting

D. as a protection of

E. to protect

 

 

1.       修饰
            
(一级排除A,B,D, E )

句末的分词逻辑主语考虑; 不定式不能单独在句末做修饰成分、

the participle protecting begins a phrase that explains what the shields did

句末分词短语的逻辑主语优先考虑句子的主语(最后一句显然是要修饰盾牌的,除了c之外,其他的都修饰不到这儿)

 

2.单复数

Choices A and B awkwardly use the singular word method to refer to items of military equipment rather than to the use of such items. (单复数问题)

3.词用法

a method of 正确,,, a method to / a method doing 不正确

a method of protecting would be more idiomatic than a method to protect in A or a method protecting in B

4. 同位语:
            
同位语要考虑是否逻辑意思内涵一致

a protection in D has no noun for which it can logically substitute.


作者: elinaqu    时间: 2007-8-10 13:34

40. In metalwork one advantage of adhesive-bonding over spot-welding is that the contact, and hence the bond­ing, is effected continuously over a broad surface instead of a series of regularly spaced points with no bonding in between.

A. instead of

B. as opposed to

C. in contrast with

D. rather than at

E. as against being at

 

1 比较结构(并列)
            
(一级排除A,B,C )

must describe the effects in parallel terms

介宾结构的比较:
            
前后要补出介词。判断依据是前面的介词是否可以用在后面的名词上。 at the point 不能用 over the point

over a broad surface rather than
                at a series

2. 简洁(二级排除E )

, as against being is a wordy and unidiomatic way to establish the intended contrast.

3. 词法:

注意介词at的用法,点用at.

rather than instead of的区别
                


作者: elinaqu    时间: 2007-8-10 13:35

41. Under a provision of the Constitution that was never applied. Congress has been required to call a conven­tion for considering possible amendments to the document when formally asked to do it by the legisla­tures of two-thirds of the states.

 

A. was never applied, Congress has been required to call a convention for considering possible amendments to the document when formally asked to do it

B. was never applied, there has been a requirement that Congress call a convention for consider­ation of possible amendments to the document when asked to do it formally

C. was never applied, whereby Congress is required to call a convention for considering possible amendments to the document when asked to do it formally

D. has never been applied, whereby Congress is required to call a convention to consider possible amendments to the document when formally asked to do so

E. has never been applied. Congress is required to call a convention to consider possible amend­ments to the document when formally asked to do so

 

1
            
句子的完整性(一级排除CD )

Whereby 是个从属连词, 句子结构:
            
加上连词 whereby 造成没有主句

By introducing the subordinating conjunction whereby, C and D produce sentence fragments. è无主句

2. 时态
            

            
现在时
            

            
过去时
            
不能统一(二级排除A,B )

3. 表示目的用不定式 call … to do call .. for doing 要好

4. 代词:
            
指代动作的时候
            
do so , 不用 do it.

uses of a pronoun (it) with no noun referent.

5. There has been 不好


作者: elinaqu    时间: 2007-8-10 13:37
以下是引用caicaichang在2007-8-10 11:13:00的发言:
elinaqu!加油哦我相信你可以坚持下来的!

谢谢MM鼓励,我一定会坚持的!
作者: elinaqu    时间: 2007-8-10 18:13

42. The current administration, being worried over some foreign trade barriers being removed and our exports failing to increase as a result of deep cuts in the value of the dollar, has formed a group to study ways to sharpen our competitiveness.

A. being worried over some foreign trade barriers being removed and our exports failing

B. worrying over some foreign trade barriers being removed, also over the failure of our exports

C. worried about the removal of some foreign trade barriers and the failure of our exports

D. in that they were worried about the removal of some foreign trade barriers and also about the failure of our exports

E. because of its worry concerning the removal of some foreign trade barriers, also concerning the failure of our exports

 

1修饰
            
(一级排除A,D,E )

 分词结构比介宾结构修饰要简洁
            
being 不好;in that they were worried,前面多余

1.       平行结构(一级排除B )

 名词对名词the removal... and the failure ...

2.       习语:

 worry about & worry over

4.代词:
            
指代的数的一致

they does not agree with the singular noun administration.

The dollarè
        
特指一种货币, 不是one dollar, 2 dollars

作者: elinaqu    时间: 2007-8-10 18:14

43. In the minds of many people living in England, before Australia was Australia, it was the antipodes, the opposite pole to civilization, an obscure and unimagin­able place that was considered the end of the world.

A. before Australia was Australia, it was the antipodes

B. before there was Australia, it was the antipodes

C. it was the antipodes that was Australia

D. Australia was what was the antipodes

E. Australia was what had been known as the antipodes

 

1代词:代词不能指向虚词
                (
一级排除B,C)
                

In B, it has no logical referent, because the previous clause describes a time when there was no Australia. ?????

时间状语从句的主语和主句的主语一致,如果都是代词造成无法代替

2. 简洁( 二级排除D,E)
                 

A was what was B what was 可以省略; there was 不好

3. 平行结构:时态平行

4. 保持原句意思, ==
            
时间状语不能省略

 

又一个关于what was的问题?


作者: elinaqu    时间: 2007-8-10 18:14

44. Using a Doppler ultrasound device, fetal heartbeats can be detected by the twelfth week of pregnancy.

A. Using a Doppler ultrasound device, fetal heart-beats can be detected by the twelfth week of pregnancy.

B. Fetal heartbeats can be detected by the twelfth week of pregnancy, using a Doppler ultrasound device.

C. Detecting fetal heartbeats by the twelfth week of pregnancy, a physician can use a Doppler ultrasound device.

D. By the twelfth week of pregnancy, fetal heartbeats can be detected using a Doppler ultrasound device by a physician.

E. Using a Doppler ultrasound device, a physician can detect fetal heartbeats by the twelfth week of pregnancy.

 

1修饰:逻辑主语和主句主语不一致
            
(一级排除A,B
                

, it modifies heartbeats, the nearest free noun in the main clause; that is, choice A says that the heartbeats are using the Doppler ultrasound device

 前置和后置状语的逻辑主语和主句的主语一致,考察是否合理

 

2句子中心:(二级排除C

改变分词成主句,造成句子中心改变,而且扰乱了逻辑关系。
            
前置状语的动作通常发生在主句之前。

the wording in choice C suggests that physicians can use a Doppler ultrasound device after they detect fetal heartbeats

 

D为什么不对????

choice D the phrase using ... device should follow physician, the noun it modifies

 

By the twelfth week of pregnancy, fetal heartbeats
                can be detected using a Doppler ultrasound
                device by a physician

By the twelfth week of pregnancy, fetal heartbeats
                can be detected
by a physician using a Doppler ultrasound
                device


作者: elinaqu    时间: 2007-8-10 18:14

45. Delighted by the reported earnings for the first quarter of the fiscal year, it was decided by the company manager to give her staff a raise.

A. it was decided by the company manager to give her staff a raise

B. the decision of the company manager was to give her staff a raise

C. the company manager decided to give her staff a raise

D. the staff was given a raise by the company manager

E. a raise was given to the staff by the company manager

1 修饰:( 一级排除 A,B,D,E )
                

前置状语的逻辑主语合理
                

2.被动语态:
                

无理由的被动语态不采用


作者: elinaqu    时间: 2007-8-10 18:15

46. A study commissioned by the Department of Agricul­ture showed that if calves exercise and associated with other calves, they will require less medication and gain weight quicker than do those raised in confinement.

A. associated with other calves, they will require less medication and gain weight quicker than do

B. associated with other calves, they require less medication and gain weight quicker than

C. associate with other calves, they required less medication and will gain weight quicker than do

D. associate with other calves, they have required less medication and will gain weight more quickly than do

E. associate with other calves, they require less medication and gain weight more quickly than

 

1.修饰-- 副词修饰形容词(一级排除A,B,C

the adverbial phrase more quickly than to modify the verb phrase gain weight.
                

In A, B, and C, quicker than is incorrect because an adjective should not be used to modify a verb phrase(考试的时候能迅速找出选项差异才能分辨出这个3-2split, 较隐含的3-2split).

 

2.平行结构(一级排除A,B,C,D

 时态一致

表事实宾语从句可以用一般现在时,本题主要是时态要一致。

3. 比较结构
                

A主语的比较
            
此处
            
省略了后面的谓宾结构  ????

B
            
比较词的选择  v+ more + adv. + than ;  adv. 修饰 V. 合理

 对主系表的句型主语比较:

A is/has + adj. than B …. AB为对等名词, 此时
            
不用补出
            
系动词。

对主谓宾全的句型主语比较:

A do sth than B do.AB为对等名词,加do是为了显示这是一个可以发出动作的名词,因而确定是主语,进而确定是主语对比)


作者: elinaqu    时间: 2007-8-10 18:15

47. Displays of the aurora borealis, or "northern lights," can heat the atmosphere over the arctic enough to affect the trajectories of ballistic missiles, induce electric currents that can cause blackouts in some areas and corrosion in north-south pipelines.

A. to affect the trajectories of ballistic missiles, induce

B. that the trajectories of ballistic missiles are affected, induce

C. that it affects the trajectories of ballistic missiles, induces

D. that the trajectories of ballistic missiles are affected and induces

E. to affect the trajectories of ballistic missiles and induce

1. 习语
            
enough to do sth.
                
(一级排除B,C,D

can heat... enough to affect in A and E is more idiomatic than the use of the subordinate clause beginning with that

 

2.平行结构(二级排除A,

数的一致和时态的一致

平行结构只有两个的时候必须用连词,不能只用
            
逗号。 AB==>and B

Induceè带来

Affectè影响

两个都是抽象动作

 

3. 单复数一致

C lacks agreement in using the singular pronoun it to refer to the plural noun displays;

4.句子完整性and D is faulty because induces cannot fit grammatically with any noun in the sentence. è没有动词发起者


作者: elinaqu    时间: 2007-8-10 18:15

48. The golden crab of the Gulf of Mexico has not been fished commercially in great numbers, primarily on account of living at great depths-- 2,500 to 3,000 feet down.

A. on account of living

B. on account of their living

C. because it lives

D. because of living

E. being they live

 

1.代词指代, 数的一致(一级排除B,E

2. 词法 :  (because句子优选) (二级排除A,D

A. because + 完整的句子

B. on account of / because of + n. ==
            
此处没有明确的限定名词的内涵

C. Because on account of / because of

because, which appears in C and E, is preferable here since because can introduce a complete subordinate clause explaining the reason why the golden crab has not been fished extensively

. C, which uses because and it as the singular subject of a clause, is the best choice.

(言下之意,主谓宾说的最清楚

3.一般现在式表示持续发生的事实和真理


作者: elinaqu    时间: 2007-8-10 18:16

49. The cameras of the Voyager II spacecraft detected six small, previously unseen moons circling Uranus, which doubles to twelve the number of satellites now known as orbiting the distant planet

A. which doubles to twelve the number of satellites now known as orbiting

B. doubling to twelve the number of satellites now known to orbit

C. which doubles to twelve the number of satellites now known in orbit around

D. doubling to twelve the number of satellites now known as orbiting

E. which doubles to twelve the number of satellites now known that orbit

 

1非限制性定语从句(一级排除A,C,E

非限制性定语从句 which 指向前面的名词(这里是不是指前面的这件事所以不能用which?)

现在分词在句末表示伴随

2.词法
            
A as B,A/B 对应。
                Known to orbit
known as orbiting … 好。


作者: elinaqu    时间: 2007-8-10 18:16

50. As a baby emerges from the darkness of the womb with a rudimentary sense of vision, it would be rated about 20/500. or legally blind if it were an adult with such vision.

A. As a baby emerges from the darkness of the womb with a rudimentary sense of vision, it would be rated about 20/500, or legally blind if it were an adult with such vision.

B. A baby emerges from the darkness of the womb with a rudimentary sense of vision that would be rated about 20/500, or legally blind as an adult

C. As a baby emerges from the darkness of the womb, its rudimentary sense of vision would be rated about 20/500; qualifying it to be legally blind if an adult

D. A baby emerges from the darkness of the womb with a rudimentary sense of vision that would be rated about 20/500; an adult with such vision would be deemed legally blind.

E. As a baby emerges from the darkness of the womb, its rudimentary sense of vision, which would deemed legally blind for an adult, would be rated about 20/500.

 

1.       分号的使用:(一级排除C
                

分号前后都必须是成分完整的句子

 

2.       代词:(二级排除A,B,E
            
从句主句的逻辑主语一致

In choice A, it, the subject of the main clause, seems to refer to baby,当有代词时。考虑其逻辑意思的合理性

Similarly, choices B and E use awkward and ambiguous phrasing that suggests that the sense of vision, rather than an adult with 20/500 vision, would be considered legally blind


作者: yukee820    时间: 2007-8-11 00:21

楼主总结的真好!!!加油!

我就跟着你的进度复习了! 谢谢


作者: gyan    时间: 2007-8-11 00:36

为啥不总结OG11呢?OG10不是很老的题?


作者: elinaqu    时间: 2007-8-11 22:32

51. While Jackie Robinson was a Brooklyn Dodger, his courage in the face of physical threats and verbal attacks was not unlike that of Rosa Parks, who refused to move to the back of a bus in Montgomery, Alabama.

A. not unlike that of Rosa Parks, who refused

B. not unlike Rosa Parks, who refused

C. like (Rosa Parks and her refusal

D. like that of Rosa Parks for refusing

E. as that of Rosa Parks, who refused

 

1
            
比较结构:
            
比较结构对象合理(一级排除B,C

: in B, Jackie Robinson's courage is compared to Rosa Parks herself, not to her courage, and in C it is compared to both Rosa Parks and her refusal

 

2.逻辑意思保持(二级排除D,E

not unlike  不等于 like / as 
            
从句改变成并列句会造成句子意思改变。

 

3. 修饰(二级排除D

for …介词短语的逻辑主语优先指向句子主语。

Choice D does not make clear whether it was Jackie Robinson or Rosa Parks who showed courage in refusing to move to the back of the bus(逻辑意思表达的准确性,介词短语做状语到底修饰谁?); in fact, saying for refusing rather than who refused makes it sound as if courage moved to the back of the bus (for 短语做主语应该优先修饰句子主语).

 

as like的区别
                

1. 表示象...一样时, as接从句, like 接短语
                

Do everything as I do. 象我一样做。
He is/looks like his mother.
他长得像他妈妈。
                

2. as当介词接短语时,表示作为...不是象...一样的意思.like 当动词时,是喜欢的意思,不要搞混为象...

We should study as Lenin studied. 我们应该像列宁那样学习。
As a League Member,I should take everything in the lead.
作为一名团员,我应该起带头作用。
                


作者: elinaqu    时间: 2007-8-11 22:34

52. The rising of costs of data-processing operations at many financial institutions has created a growing opportunity for independent companies to provide these services more efficiently and at lower cost.

A. The rising of costs

B. Rising costs

C. The rising cost

D. Because the rising cost

E. Because of rising costs

 

1. 句子完整性(一级排除D, E
                

出现连词引导的分句一定要有对应的主句。

Choices D and E produce sentence fragments since Because makes the clause subordinate rather than independent.

 

2. 主谓一致(一级排除B,E

单复数

 

1.       逻辑意思(二级排除A

从中心词看哪个意思表达最合理The rising 还是 cost 提供了机会造成。。。

The rising cost / The rising of costs has created a growing opportunity for independent companies to provide these services more efficiently and at lower cost.

Has created 表示前面那个动作已经完成了, 完成的完整的过程, 不能用动名词the rising of costsè gerund is only the name of an action. 要么用the rising cost, 核心词在cost上面, a growing opportunity并列, 要么用the rise in costè
            
但是就不平行了

 

4. 平行结构:感觉文中有大并列的感觉:the rising cost……a growing opportunity

 


作者: elinaqu    时间: 2007-8-11 22:49

53. There is no consensus on what role, if any, is played by acid rain in slowing the growth or damaging forests in the eastern United States.

A. slowing the growth or damaging

B. the damage or the slowing of the growth of

C. the damage to or the slowness of the growth of

D. damaged or slowed growth of

E. damaging or slowing the growth of

 

1.       平行结构(一级排除A,B

逻辑意思平行放在第一位;抽象名词和具体名词不能对应。A.  the growth forest 不平行

形式平行要仔细考虑,不是光看到有
                –ing
或者
                –ed
就可以。。

 

2.       词法(二级排除C,D

in doing sth 来表达在什么方面做某事,类似的结构多次出现

ETS非常喜欢的一个结构:用 in doing sth 来表达在什么方面做某事,类似的结构多次出现

 

3.       动词和名词的区别.

名词常常表达意思不准确且wordy,动作有执行对象,常常表达的比较准确。

 

4.       逻辑意思

介词+ v+ ed + 名词 , 过去分词是形容词性,重点放在后面的名词上了;

介词+V+ing + n. 重点强调前面的动名词

记住e正确答案中这样的并列是允许的(看意思,不要仅仅停留的表面形式上)

in slowing the growth of forests and damaging the growth of forests

 


作者: elinaqu    时间: 2007-8-11 22:54

54. Galileo was convinced that natural phenomena, as manifestations of the laws of physics, would appear the same to someone on the deck of a ship moving smoothly and uniformly through the water as a person standing on land.

A. water as a

B. water as to a

C. water; just as it would to a

D. water, as it would to the 

E. water; just as to the

 

1.       平行结构;
            
(一级排除A,C,D,E

 the same to X as to Y ; 形式平行;
            
对象范围的平行: someone / a person 泛指
            

            
泛指; someone / the person 不平行。

2. 代词:
            
代词指代的单复数平行

3.分号:
            
分号后面需要是完整的句子。

4.冠词
                a/ the

                


作者: elinaqu    时间: 2007-8-11 23:00
以下是引用yukee820在2007-8-11 0:21:00的发言:

楼主总结的真好!!!加油!

我就跟着你的进度复习了! 谢谢

谢谢mm。欢迎补充,加油


作者: elinaqu    时间: 2007-8-11 23:30
以下是引用gyan在2007-8-11 0:36:00的发言:

为啥不总结OG11呢?OG10不是很老的题?

我个人觉得OG10的语法解释好一些,而且不会的题可以找到nn门的讨论贴


作者: elinaqu    时间: 2007-8-12 22:18

 

55. A recent study has found that within the past few years, many doctors had elected early retirement rather than face the threats of lawsuits and the rising costs of malpractice insurance.

(A)  had elected early retirement rather than face

(B)  had elected early retirement instead of facing

(C)  have elected retiring early instead of facing

(D)  have elected to retire early rather than facing

(E)   have elected to retire early rather than face

 

1
            
时态(一级排除A,B,C

within the past years 做状语表示从过去持续到现在的动作,主句的时态要用完成时。
            
已经完成的动作用完成时态不能还继续表示进行时态的意思。注意和现在完成进行时态区分。
            
主要从逻辑意思进行区别:已经选择一个东东, 不是已经选择一个动作

C is faulty because have elected, which is correct in tense, cannot idiomatically be followed by a

participle such as retiring .this is very important. It means we cannot both finish sth and at the same time, identify this same sth as ongoing….

 

1.       平行结构(二级排除D
                

名词/ 动词 / 分词
            
不能并列

2.       优先结构:
                

rather than > instead of

 

 


作者: elinaqu    时间: 2007-8-12 22:19

56. Architects and stonemasons, huge palace and temple clusters were built by the Maya without benefit of the wheel or animal transport.

A.      huge palace and temple clusters were built by the Maya without benefit of the wheel or animal transport

B.      without the benefits of animal transport or the wheel, huge palace and temple clusters were built by the Maya

C.      the Maya built huge palace and temple clusters without the benefit of animal transport or the wheel

D.     there were built, without the benefit of the wheel or animal transport, huge palace and temple clusters by the Maya

E.      were the Maya who, without the benefit of the wheel or animal transport, built huge palace and temple clusters

 

1.       同位语结构(一级排除A,B,D

逻辑对象要对应;
            
GMAT 的优选结构。主语和谓语之间, 除非是插入语, 不能加逗号.

 

2.       平行结构(二级排除E

系表结构的对应Architects and stonemasons, were the Maya ; 泛指
            

            
特指
            
不对

E is awkwardly phrased and produces a sentence fragment, because the appositive noun phrase Architects and stonemasons cannot serve as the subject of were the Maya. è主语和谓语之间, 除非是插入语, 不能加逗号.

主语, 插入语, 谓语.

主语, 介词短语, 分词短语, 定语从句, 谓语.

同位语, 主语谓语.

 

E即使没有语法错误,也不能选,因为它 awkward; 这就是gmat的精髓。(简洁,清晰)然而, 造成E awkward的原因是:,without the benefit of …. 是修饰动词的, 放在主谓之间, 不好, 要放在谓语的后面, 或者句首. 谓语动词象一座山

某人是作…..的某人=è这样的描述没有直接说:  某人作某事
            
来得流畅清楚.

 

3.       被动语态要避免。

4.       简洁

 


作者: elinaqu    时间: 2007-8-12 22:59

57. In astronomy the term "red shift" denotes the extent to which light from a distant galaxy has been shifted toward the red, or long-wave, end of the light spec­trum by the rapid motion of the galaxy away from the Earth.

(A)  to which light from a distant galaxy has been shifted

(B)   to which light from a distant galaxy has shifted

(C)  that light from a distant galaxy has been shifted

(D)  of light from a distant galaxy shifting

(E)   of the shift of light from a distant galaxy

 

1.被动语态(一级排除B,D,E

  by …. 是被动语态的标志。
                

所有的选项都受到 by the rapid motion of the galaxy away from the earth. 的影响。
                    

 

2. 习语:(二级排除C

denote the extent to …

In C, the construction the extent that light is ungrammatical; denotes the extent must be completed by to which.
            

 


作者: elinaqu    时间: 2007-8-12 23:04

58. William H. Johnson's artistic debt to Scandinavia is evident in paintings that range from sensitive portraits of citizens in his wife's Danish home, Kerteminde, and awe-inspiring views of fjords and mountain peaks in the western and northern regions of Norway.

(A)   and

(B)   to

(C)   and to

(D)  with

(E)   in addition to

1.       习语一级排除A, C,D,E

range from X to Y

is evident in paintings 方面明显


作者: elinaqu    时间: 2007-8-12 23:26

59. In 1978 only half the women granted child support by a court received the amount awarded; at least as much as a million and more others had not any support agreements whatsoever.

(A)  at least as much as a million and more others had not any

(B)  at least as much as more than a million others had no

(C)  more than a million others had not any

(D)  more than a million others had no

(E)   there was at least a million or more others without any

1比较结构:一级排除A, B

at least as much as  / more than 意思重复

修饰:B incorrectly use much rather than many to describe the countable noun others much+ 不可数; many + 可数

2. 简洁二级排除E

 there be 结构不太好

 单复数it incorrectly links the singular verb was with the plural subject others.

3. 习惯用法
            
二级排除C

not any 没有
            
直接用 no 简洁. In A and C, not any support agreements is wordy and awkward

紫禁城参观的那个, 就是用的no /not any 的简洁性来排除错误的.

The Forbidden City in Beijing, from which the emperors ruled by heavenly mandate, was a site which a commoner or foreigner could not enter without any permission, on pain of death.

(A) which a commoner or foreigner could not enter without any permission, 普通人不能进去,没有许可的话

(B) which a commoner or foreigner could enter without any permission only

(C) which no commoner or foreigner could enter without permission, 没有普通人能进去, 没有许可的话

(D) which, without permission, neither commoner or foreigner could only enter,C

(E) which, to enter without permission, neither commoner or foreigner could do,

1、语气上C更好

2without any permission是一种wordy的表达方式
                

从简洁性上考虑得更多。

 

Although all the proceedings of the Communist party conference held in Moscow were not carried live, Soviet audiences have seen a great deal of coverage.

(A) all the proceedings of the Communist party conference held in Moscow were not carried live

(B) all the Communist party conference’s Moscow proceedings were not carried live

(C) all the Communist party conference Moscow proceedings have not been carried alive

(D) not all the Communist party conference Moscow proceedings have been carried aliveE

(E) not all the proceedings of the Communist party conference held in Moscow were carried live


作者: elinaqu    时间: 2007-8-12 23:52

60. According to a recent poll, owning and living in a freestanding house on its own land is still a goal of a majority of young adults, like that of earlier generations.

(A)  like that of earlier generations

(B)   as that for earlier generations

(C)  just as earlier generations did

(D)  as have earlier generations

(E)   as it was of earlier generations

 

1. Like/As
            
比较问题:
             as +
句子; like + n. 一级排除B,C,D

根据逻辑意思决定比较的对象是单独比较一个名词还是比较一种状态。

like + n. ; as+ 完整句子

Like+ n, 通常放在句首或者句中,但是不肯定。
            
放在句末也是优先指向主语。

As 的比较:
                

--As结构成为正确答案

在这样的情况下,as是作为连词出现的。既然是连词就只能带一个句子。但是由于as所带的句子与主句有很多相同的地方,所以可以使用加助动词省略的形式成为正确答案。比如:

A do ***, as B do

A can *** as B can

A is *** as B and C are

补出助动词的原则是必须和主句的类型一致,而且要时态一致。

但是如果某个as结构自己就带了时间状语,那么该as结构的时态和自己带的状语保持一致,而类型和主句保持一致。

A do *** as B did ten years ago.

二、让Like结构成为正确答案

在这样的情况下,Like是作为介词出现的。既然是介词,就只能带宾语。而且宾语的类型和主句主语的类型必须保持一致。

Like A’s Book, B’s Book is red.

典型的错误为:Like A, B’s book is red.(人和人比,书和书比)

三、典型的干扰选项

看到下面的选项一定不要被它们所迷惑:

similar to

contrary to(但在like/unlike题型中有可能成为正确答案)

 

2. That 的指代容易有歧义 . 排除 A
                

like 虽然放在句末,但是做状语也应该优先修饰主语(例似于with放在句末);另外that指带也有问题。

 

3.gmat 中句子从整体上看要有大并列的感觉,

it refers unambiguously to the phrasal subject owning ... land, the verb was corresponds to is, and today's young adults are appropriately compared to earlier genera­tions(大并列).
            

gmatthat一般不能单独指代,要有修饰成分,如that of ...that in....等。it一般不存在过去和现在的区分,如than it was five years ago等,但地点是要区分的,这好像是你近来的一个总结。不知我理解的对不对。


作者: yukee820    时间: 2007-8-15 03:30

楼主加油啊!!!希望可以在考试前看到这个帖子的完整版


作者: elinaqu    时间: 2007-8-16 15:01
又来了
[此贴子已经被作者于2007-8-22 10:18:03编辑过]

作者: elinaqu    时间: 2007-8-16 15:01

61. The Gorton-Dodd bill requires that a bank disclose to their customers how long they will delay access to funds from deposited checks.

(A)
                    
that a bank disclose to their customers how long they will delay access to funds from deposited checks

(B)  a bank to disclose to their customers how long they will delay access to funds from a deposited check

(C)
                    
that a bank disclose to its customers how long it will delay access to funds from deposited checks

(D)
                    
a bank that it should disclose to its customers how long it will delay access to funds from a depos­ited check

(E)  that banks disclose to customers how long access to funds from their deposited check is to be delayed

 

1单复数一级排除A,B,D

代词的一致 ==
            
代词和指代物的一致;

代词 their/they a bank 不一致

逻辑意思的单复数 ==customers  a depos­ited check 

 2词法级排E

 require 用法
            
require that …./  require sb. To do sth….. require of sb. That sth. ……

3. 被动语态
            
prefer.


作者: elinaqu    时间: 2007-8-22 10:19

☆☆☆62. Geologists believe that the warning signs for a major earthquake may include sudden fluctuations in local seismic activity, tilting and other deformations of the Earth's crust, changing the measured strain across a fault zone, and varying the electrical properties of underground rocks.

(A)  changing the measured strain across a fault zone and varying

(B)  changing measurements of the strain across a fault zone, and varying

(C)  changing the strain as measured across a fault zone, and variations of

(D)  changes in the measured strain across a fault zone, and variations in

(E)   changes in measurements of the strain across a fault zone, and variations among

1.       平行结构:一级排除A,B,C
                

平行结构:
            
名词比较, changing 无法
            
和后面的名词平行

 fluctuations, deformations, changes, variations是平行的,如果用changes, vary没有发出动作的名词

平行结构首先从逻辑意思上判断是需要动名词还是名词进行平行。动名词的比较前面需要出现对应的逻辑主语。

 

2.比较对象:应该是strain 更加符合逻辑意思   二级排除E

3. 词法in/among二级排除E

among 表示
            
其后接的事物有好几种不同的,在这几个不同的事物中比较。 In 表示
            
在后面的事物中有好几个子事物(特性,子集)中
            
比较among in E wrongly suggests a comparison of different electrical properties rather than of different behaviors of the same properties.

 

4
            
所有格:    A of B 强调的是 A
            
要比较意思

 


作者: elinaqu    时间: 2007-8-22 10:19

63. Health officials estimate that 35 million Africans are in danger of contracting trypanosomiasis, or "African sleeping sickness," a parasitic disease spread by the bites of tsetse flies.

(A)  are in danger of contracting(习语)

(B)  are in danger to contract

(C)  have a danger of contracting

(D)  are endangered by contraction

(E)   have a danger that they will contract

 

1习语: be in danger of sth.
            
一级排除B,D,E

2
            
时态:二级排除C

现在完成时没有理由,而且从逻辑意思上看 have a danger 也不对。

the passive construction in D would be unnecessarily wordy and also imprecise, because it is the disease more than the act of contracting it that poses the danger.(意思表达的细微差别) èbe endangered by的意思也有区别, 往往表示环境/野生动物被endangered


作者: elinaqu    时间: 2007-8-22 10:19

64. Unlike a funded pension system, in which contributions are invested to pay future beneficiaries, a pay-as-you-go approach is the foundation of Social Security.

(A)  a pay-as-you-go approach is the foundation of Social Security

(B)   the foundation of Social Security is a pay-as-you-go approach

(C)  the approach of Social Security is pay-as-you-go

(D)  Social Security's approach is pay-as-you-go

(E)   Social Security is founded on a pay-as-you-go approach

1.比较结构 unlike A , B A/B要对等。一级排除A,B,C,D

compares one kind of system of providing for retirees, the funded pension system, with another such system. Social Security这里就是说social security本身就是一种system
                

2.When, where, in which 的用法:
1. when , where
可以用在非限制性關係子句中
ex: He came from
Herne
                
Bay
, where Lally had once spent a holiday.
2. when , where
也可以用在限制性關係子句中,但前面必須有特定的名詞. 且像" Lodon" 這種專有名詞的地名後面是不跟限制性關係子句的.
ex: This is the street where I live.
3.
在限制性關係子句中,
time: 可用 at which 替代 when
place, room, street, year, month: 可用in which 替代 where when
ex: the place in which they found themselves.
day 之後可用on which 替代 when
situation, stage, arrangement, system 之後, 可用 in which 替代 where, whereby.

所以, in which 常用在限制性關係子句中, (簡單看就是前面不要有 ,)可以替代 where, when
可以替代
1.
時間觀念
year, month
2.
地點
place, room, street,
3.
某種情境
situation, stage, state, arrangement, system
in which
用在非限制性關係子句中, (簡單看就是 ... , in which...)
Ex: Sometimes you may feel too frail to cope with things, in which cases do them as soon as it is convenient.
表示要對所說的事情進行補充時, 可用
            
介詞+which+名詞
            
這種句法
讓我大膽結論一下:..., in which+ 非名詞
            
必錯. 例如大全149
..., in which +名詞 :名詞+in which ,則與 when or where 都可以互換. 必須找其他錯誤.
此外,that which ...,where 的用法, 在考古題中沒有對過!


作者: elinaqu    时间: 2007-8-22 10:20

65. Critics of the trend toward privately operated prisons consider corrections facilities to be an integral part of the criminal justice system and question if profits should be made from incarceration.

(A)  to be an integral part of the criminal justice system and question if

(B)   as an integral part of the criminal justice system and they question if

(C)  as being an integral part of the criminal justice system and question whether

(D)  an integral part of the criminal justice system and question Whether

(E)   are an integral part of the criminal justice system, and they question whether

 

1.      词法
            
consider + n. + n.
            
一级排除A,B,C,E

When consider means "regard as," as it does in this sentence, its object should be followed immediately by the phrase that identifies or describes that object

Consider sth sth/adj

Make sth sth/adj

Cause sth to do sth

Be believed to do

Be expected to do

Be estimated to do

Be regarded as

Be credited with

2.  Whether/ if
            
…..//// whether
表示是否。

3.  平行结构
            
主语相同的时候
            
后面的一个要省略。

没有新的主语, 两个动词的发起一样的时候, 只要一个名词发起者就可以了

A do b and A do c. èèmust be altered into:

A do b and do c. 

or even

A do and do X.

 


作者: elinaqu    时间: 2007-8-22 10:21

66. The Federal Reserve Board's reduction of interest rates on loans to financial institutions is both an acknowledgment of past economic trends and an effort to influence their future direction.

(A)  reduction of interest rates on loans to financial institutions is both an acknowledgment of past economic trends and an effort

(B)   reduction of interest rates on loans to financial institutions is an acknowledgment both of past economic trends as well as an effort

(C)  reduction of interest rates on loans to financial institutions both acknowledge past economic trends and attempt

(D)  reducing interest rates on loans to financial institutions is an acknowledgment both of past economic trends and an effort

(E)   reducing interest rates on loans to financial institutions both acknowledge past economic trends as well as attempt

1.平行结构一级排除B,D,E
                

A. both … and 前后要逻辑对称,符合原句的意思,不能随意更改位置。

B. 系表结构的前后平行:
            
前后的词性的平行,数的平行。

C. 抽象名词不能做句子的主语作为动作的发起者。

it is imprecise to characterize a reduction as performing actions such as acknowledging or attempting ( is 前后要并列,注意要更广泛,更深刻的去理解并列的感觉===动词的发起不能是抽象的名词啊!!!). 系表结构是一种并列

 

2.
            
词性:二级排除C     

reduction 不能做 acknowledge/ attempt 的主语,而且这样修改也改变了原句的意思

A. 动名词-强调具体动作的过程;动词的名词形式-强调动作的前提,结果的全方位的东东

B. 动名词不能做所有格的宾语。

3.单复数二级排除C

the plural verbs acknowl­edge and attempt do not agree with their singular subject, reduction


作者: elinaqu    时间: 2007-8-22 10:21

67. Congress is debating a bill requiring certain employers provide workers with unpaid leave so as to care for sick or newborn children.

(A)  provide workers with unpaid leave so as to

(B)  to provide workers with unpaid leave so as to

(C)  provide workers with unpaid leave in order that they

(D)  to provide workers with unpaid leave so that they can

(E)   provide workers with unpaid leave and

 

1.
            
词法Require二级排除A,C,E

 A. require sb. To do sth.

2.
            
词法so as to/so that二级排除B

so as to : 句子主语为其逻辑主语,考察是否合理。

E says that the bill being debated would require the employers themselves to care for the children

 

in order that GMAT里被认为是不正规的表达方式,就象instead of,但是不是绝对的,如果选项里有其他的合理的表达方式,例如SO THAT,就应该关注,但建议不把这个倾向作为最高优先级,还是先从其他方面排除。另外in order to通常也被认为是WORDY的,因为可以用TO直接表达,但是要比较选项。同时,SO AS TO的逻辑主语是句子主语,例如AB,明显出现了逻辑意思上的错误,这也是需要注意的。

如果在句子中其他没有什么可比的了,则个人认为:
so that> so ... as to > in order to > in order that

Because of / On account of / Despite/ Despite for/  Except for / Account for
这些词通常会在句首做原因状语,ETS倾向于直接加名词,不加动名词.


作者: elinaqu    时间: 2007-8-22 15:22

68. Often visible as smog, ozone is formed in the atmosphere from hydrocarbons and nitrogen oxides, two major pollutants emitted by automobiles, react with sunlight.

(A)  ozone is formed in the atmosphere from

(B)  ozone is formed in the atmosphere when

(C)  ozone is formed in the atmosphere, and when

(D)  ozone, formed in the atmosphere when

(E)   ozone, formed in the atmosphere from

 

1.句子完整性一级排除D,E

Choice D,E omits the main verb, is, leaving a sentence fragment

1.      从句和主句不能并列二级排除C,且造成句子不完整

In choice C, the use of the conjunction and results in the illogical assertion that the formation of ozone in the atmosphere happens in addition to, rather than as a result of, its formation when hydrocarbons and nitrogen oxide react with sunlight.

 

3.介词短语后面只能接名词,不能接句子。二级排除A

, from后面的东东不对In choice A, the construction from hydrocarbons and nitrogen oxides ... react is ungrammatical.

From是介词, 介词后面只能接名词, 不能接句子!!!!  

Be based on sth一样!!!og90


作者: elinaqu    时间: 2007-8-22 15:23

69. Although she had signed a pledge of abstinence while being an adolescent, Frances Willard was 35 years old before she chose to become a temperance activist.

(A)  while being an adolescent

(B)   while in adolescence

(C)  at the time of her being adolescent

(D)  as being in adolescence

(E)   as an adolescent

 

1.       词法一级排除A,B,C,D,

while/ at the time of 都只能表现时间点;
            
只有 as 可以表现一段时间。

As 表示作为, 或者: as she was an adolescent.

后接名词表示某一年龄段时,as。如:

As a young man, he was active in sports. 

While doing表示伴随, 有轻微转折味道.

Q1,while也可以接时间段。。
            
为什么不用while这里?

Q2. B选项可不可以看成是
            
省略she was

Q3, as being是不是通常都不对

我觉得这题用as的原因主要是在语法书里有一点as的用法后接名词表示某一年龄段时,as:As a young man, he was active in sports. 


作者: elinaqu    时间: 2007-8-22 15:23

70. A President entering the final two years of a second term is likely to be at a severe disadvantage and is often unable to carry out a legislative program.

(A)  likely to be at a severe disadvantage and is often unable to

(B)   likely severely disadvantaged and often unable to

(C)  liable to be severely disadvantaged and cannot often

(D)  liable that he or she is at a severe disadvantage and cannot often

(E)   at a severe disadvantage, often likely to be unable that he or she can

1.       平行结构
            
一级排除 B C D E

     后面的系动词补出表示平行。原题and后补出is 是为了更清楚的指示谁和谁并列。

3.       词法 

can not often/ often unable to

cannot often意思是不是每次都能

is often unable to意思是没有能力

 

Be likely to do sth

It is unlikely to do
            
是错误的
            
不可以是that

 

Disadvantaged at a disadvantage的区别.
            

区分 at a disadvantage处于不利的位置. / disadvantaged having social problems 的意思差别

 

In B and C, disadvantaged, which often means "hampered by substandard economic and social conditions," is less precise than at a disadvantage处于不利的位置.

Disadvantaged: having social problems, such as a lack of money or education, which make it difficult for you to succeed

disadvantaged areas of the city

 

副词的位置不能轻易改变,也就是修饰对象不能变,变了就错了,不用深究意思的细微差别è
            
一定要考虑到底是修饰什么的. 意思是什么 (only the husband, expect almost to )


[此贴子已经被作者于2007-8-23 9:57:15编辑过]

作者: elinaqu    时间: 2007-8-22 15:24

71. The original building and loan associations were organized as limited life funds, whose members made monthly payments on their share subscriptions, then taking turns drawing on the funds for home mortgages.

(A)  subscriptions, then taking turns drawing

(B)  subscriptions, and then taking turns drawing

(C)  subscriptions and then took turns drawing

(D)  subscriptions and then took turns, they drew

(E)   subscriptions and then drew, taking turns

 

1
            
平行结构一级排除A,B,E
                

表示时间上先后顺序以及带有因果关系的
            
and

 

2 run-on 结构级排除D,

The wording in D results in a run-on sentence and does not specify what the members took turns doing.

3. 分词做伴随状语就不要使用 then.

分词短语作状语就不要thenè subscriptions, taking turns drawing

 

Take turns: 轮流,依次。Draw on, 利用


作者: elinaqu    时间: 2007-8-22 15:25

☆☆☆72. The number of undergraduate degrees in engineering awarded by colleges and universities in the United States
                increased by more than twice from 1978 to 1985.

(A)  increased by more than twice

(B)   increased more than two times

(C)  more than doubled

(D)  was more than doubled

(E)   had more than doubled

1 时态&语态:一级排除D,E

 rom 1978 to 1985 表示过去时间。

The past perfect tense in E, had... doubled, is inappropriate unless the increase in engineering degrees is specifically特定的,明确的 being viewed as having occurred further back in the past than some subsequent event. 过去完成时应该用在某事件在过去比另外一个事件发生的更早,这种时候才恰当è一定要出现明确的正确的过去时态的动作作为时间参照点)

The number of undergraduate degrees in engineering awarded by colleges and universities in the United States more than doubled from 1978 to 1985.è未划线部分的时间状语是完全正确的过去时间, 用一般过去时态不要太灵啊!!!è
            
有个business的题目, 一样的时间, from…to….

D's passive verb was ... doubled suggests without warrant that some unnamed agent increased the number of engineering degrees.

被动语态)è怎样算with warrant: 出现by sb/ by sth.

特别是在未划线部分出现的话, 就很可能是用被动语态.

 

 

2
            
词法:二级排除A,B

Atwice只能作副词, 在句子中表达的意思却是比较, 这样的题目, 首先要弄清楚比较的双方是否合理, 如果合理的话, 再看twice作为副词的位置: 可以用在as..as或比较级之前, 也可以放在比较点的名词之前, 担要保证twice前后的词要能直接搭配(副词的作用). 不能做名词词性接在介词后面 by …Choice A is faulty because an adverb such as twice cannot function as an object of the preposition by(用词). BY 后面要接名词
                by 10 percent

 

 B. , 倍數辭句型的用法敘述: .. times the/my N ... 結構︰倍數詞+the(或所有格)+名詞說明︰此句型意為的幾倍<倍數詞><名詞>之間一定有 the <所有格>。有時可改為“<倍數詞>as<形容詞>”。三倍以上的,通常用“times”。而“more than<倍數詞>the(或<所有格>)+<名詞>”譯為的幾倍還不止 It was over twice the distance of the earth from the sun. 那是地球到太陽的距離的兩倍多。

 

C. two times 表示两次; double 表示两倍。B增长了超过两次 distorts the sentence's meaning, stating that the number of engineering degrees conferred increased on more than two distinct occasions.(歧义)
            
增长了两次!!!


作者: elinaqu    时间: 2007-8-22 15:27

73. The British Admiralty and the War Office met in March 1892 to consider a possible Russian attempt to seize Constantinople and how they would have to act militarily to deal with them.

(A)  how they would have to act militarily to deal with them

(B)   how to deal with them if military action would be necessary

(C)  what would be necessary militarily for dealing with such an event

(D)  what military action would be necessary in order to deal with such an event

(E)   the necessity of what kind of military action in order to take for dealing with it

 

 

1. 代词
            
一级排除A,B

In choices A and B, the pronoun them has no antecedent;

通常一个概括性的名词要比代词清晰的多(这往往成为正确答案的标志)

2.简洁
            
二级排除C. E

would be necessary militarily / military action would be necessary

3.并列Consider sth.名词.+sth.名词.

the sentence states that the Admiralty and the War Office met to consider x and y, where x is the noun phrase a possible Russian attempt. D provides a noun phrase, military action, that matches the structure of x more closely than do the corresponding noun elements in the other choices.

4.虚拟语气

furthermore, the if clause in B must take should rather than would (虚拟语气)


作者: elinaqu    时间: 2007-8-22 15:27

74. Growing competitive pressures may be encouraging auditors to bend the rules in favor of clients; auditors may, for instance, allow a questionable loan to remain on the books in order to maintain a bank's profits on paper.

(A)  clients; auditors may, for instance, allow

(B)  clients, as an instance, to allow

(C)  clients, like to allow

(D)  clients, such as to be allowing

(E)   clients; which might, as an instance, be the allowing of

 

1 词法:一级排除B,C, D

 x as an instance of y / for instance

 Like 不能用来举例

Such as to be

2.句子完整:二级排除E

分号后面是一个完整的句子   

修饰: which 指代


作者: elinaqu    时间: 2007-8-22 15:28

75. If the proposed expenditures for gathering information abroad are reduced even further, international news reports have been and will continue to diminish in number and quality.

(A)  have been and will continue to diminish

(B)  have and will continue to diminish

(C)  will continue to diminish, as they already did,

(D)  will continue to diminish, as they have already,

(E)   will continue to diminish

 

1虚拟语气(时态)
            
一级排除A,B

if x happens, y will happen.

2. 简洁/词法二级排除C, D

will continue 有表示从过去发生持续到将来的动作。加入as 从句
            
不简洁

C and D conclude with faulty as clauses that are awkward and unnecessary, because will continue describes an action begun in the past.è这个是好灵啊!!! 用词最少, 意思最好
                


作者: elinaqu    时间: 2007-8-22 15:28

76. Gall's hypothesis of there being different mental functions localized in different parts of the brain is widely accepted today.

(A)  of there being different mental functions localized in different parts of the brain is widely accepted today

(B)  of different mental functions that are localized in different parts of the brain is widely accepted today

(C)  that different mental functions are localized in different parts of the brain is widely accepted today

(D)  which is that there are different mental functions localized in different parts of the brain is widely accepted today

(E)   which is widely accepted today is that there are different mental functions localized in different parts of the brain

 

 

1. 介词短语:一级排除A,B

of there being  。。。
            
不简洁.  N.+ Of A that B / n. that A B 表达强调的重点不一样。

2.简洁/定语从句二级排除D,E

限制性定语从句用 that

Choices D and E are awkward and wordy, and both use which where that would be the preferred pronoun for introducing a clause that states Gall's point.

3.句子意思
            
主句不能随便变成从句

Further, the phrasing of E misleadingly suggests that a distinction is being made between this hypothesis and others by Gall that are not widely accepted today. è限制性定语从句带来的意思的区别è部分和整体的关系






欢迎光临 ChaseDream (https://forum.chasedream.com/) Powered by Discuz! X3.3